Re: [gmx-users] Optimizing a parallel simulation

2009-10-07 Thread Mark Abraham

vivek sharma wrote:

Hi Mark,
Thanks for your quick response.

2009/10/8 Mark Abraham >


vivek sharma wrote:

Hi There,
While running a parallel MD simulation, I got following message
while playing with parameters:
NOTE 3 [file aminoacids.dat, line 1]:
 The optimal PME mesh load for parallel simulations is below 0.5
 and for highly parallel simulations between 0.25 and 0.33,
 for higher performance, increase the cut-off and the PME grid
spacing
---

I don't have any idea when a run should be called a highly
parallel simulation?


Well below 224 :-)


Is it depend on problem-size or the number of processor we are
using?


Processors.


I am trying to optimize my simulation to run on 224 processors
and system consist of:

Total atom count = 54640
where water molecules = 17074


You will want to tweak rcoulomb and fourier_space like the above
message says. Even so, with less than 500 atoms per
particle-particle processor you may find yourself spending more time
communicating than processing. I suspect your system may be too
small to get value from so many processors, even if you have good
network (e.g. Infiniband). You will wish to read 3.17 of the manual
to start understand the issues involved here.

Thanks for pointing me to the rel;ated section in the manual. While 
tweaking the rcoulomb and fourier_space values, what parameter need to 
be observed for checking the result?

Is it the PME load or something else?


Good question. There are some heuristics reported on this list before, 
which you can search for. Varying rcoulomb affects the absolute cost of 
the real-space part, and the accuracy of both parts of the PME 
algorithm. Varying fourier_spacing affects the cost and accuracy of the 
reciprocal-space part.


Unfortunately I'm not aware of any literature that deals rigorously with 
"how accurate is enough?" for PME.


Mark


Also, I am getting following message while running grompp
-
NOTE 1 [file md.mdp, line unknown]:
 The Berendsen thermostat does not generate the correct kinetic
energy
 distribution. You might want to consider using the V-rescale
thermostat.

Any insight in the above message will be highly appreciated.


Seems self-explanatory to me - you're using Berendsen and that might
not be the right choice. Read the manual sections about the
different thermostats.

Mark
___
gmx-users mailing listgmx-users@gromacs.org

http://lists.gromacs.org/mailman/listinfo/gmx-users
Please search the archive at http://www.gromacs.org/search before
posting!
Please don't post (un)subscribe requests to the list. Use the www
interface or send it to gmx-users-requ...@gromacs.org
.
Can't post? Read http://www.gromacs.org/mailing_lists/users.php





___
gmx-users mailing listgmx-users@gromacs.org
http://lists.gromacs.org/mailman/listinfo/gmx-users
Please search the archive at http://www.gromacs.org/search before posting!
Please don't post (un)subscribe requests to the list. Use the 
www interface or send it to gmx-users-requ...@gromacs.org.

Can't post? Read http://www.gromacs.org/mailing_lists/users.php

___
gmx-users mailing listgmx-users@gromacs.org
http://lists.gromacs.org/mailman/listinfo/gmx-users
Please search the archive at http://www.gromacs.org/search before posting!
Please don't post (un)subscribe requests to the list. Use the 
www interface or send it to gmx-users-requ...@gromacs.org.

Can't post? Read http://www.gromacs.org/mailing_lists/users.php


Re: [gmx-users] Load imbalance and Y size error

2009-10-07 Thread Mark Abraham

nikhil damle wrote:

Hi all

 I am trying to simulate a system of 3168 atoms in vacuum in PBC 
box (-d=0.9 nm). Since there are no solvent molecules, i am unable to 
neutralise the system using addition of ions (Hence system has +3 
charge). As soon as i start the MD run after putting the whole system in 
box, there is imbalanced force of 80% due to which right in the 
beginning, dynamic load balancing is turned on. But even after this, 
there is ~22% loss of performance and after certain steps MD crashes 
saying :-


"Fatal error: The Y-size of the box (4.618413) times the triclinic skew 
factor (0.866051) is smaller than the number of DD cells (2) times the 
smallest allowed cell size(2.00)"


Can anyone explain the origin of this error and how to deal with it ?


Try running with one processor to see if it's your system or the domain 
decomposition algorithm that is the problem. DD doesn't necessarily cope 
well with highly heterogeneous distributions of particles.


Mark
___
gmx-users mailing listgmx-users@gromacs.org
http://lists.gromacs.org/mailman/listinfo/gmx-users
Please search the archive at http://www.gromacs.org/search before posting!
Please don't post (un)subscribe requests to the list. Use the 
www interface or send it to gmx-users-requ...@gromacs.org.

Can't post? Read http://www.gromacs.org/mailing_lists/users.php


Re: [gmx-users] Optimizing a parallel simulation

2009-10-07 Thread vivek sharma
Hi Mark,
Thanks for your quick response.

2009/10/8 Mark Abraham 

> vivek sharma wrote:
>
>> Hi There,
>> While running a parallel MD simulation, I got following message while
>> playing with parameters:
>> NOTE 3 [file aminoacids.dat, line 1]:
>>  The optimal PME mesh load for parallel simulations is below 0.5
>>  and for highly parallel simulations between 0.25 and 0.33,
>>  for higher performance, increase the cut-off and the PME grid spacing
>> ---
>>
>> I don't have any idea when a run should be called a highly parallel
>> simulation?
>>
>
> Well below 224 :-)
>
>  Is it depend on problem-size or the number of processor we are using?
>>
>
> Processors.
>
>  I am trying to optimize my simulation to run on 224 processors and system
>> consist of:
>>
>> Total atom count = 54640
>> where water molecules = 17074
>>
>
> You will want to tweak rcoulomb and fourier_space like the above message
> says. Even so, with less than 500 atoms per particle-particle processor you
> may find yourself spending more time communicating than processing. I
> suspect your system may be too small to get value from so many processors,
> even if you have good network (e.g. Infiniband). You will wish to read 3.17
> of the manual to start understand the issues involved here.
>
Thanks for pointing me to the rel;ated section in the manual. While tweaking
the rcoulomb and fourier_space values, what parameter need to be observed
for checking the result?
Is it the PME load or something else?

Thanks,
Vivek

>
>  Also, I am getting following message while running grompp
>> -
>> NOTE 1 [file md.mdp, line unknown]:
>>  The Berendsen thermostat does not generate the correct kinetic energy
>>  distribution. You might want to consider using the V-rescale thermostat.
>> 
>> Any insight in the above message will be highly appreciated.
>>
>
> Seems self-explanatory to me - you're using Berendsen and that might not be
> the right choice. Read the manual sections about the different thermostats.
>
> Mark
> ___
> gmx-users mailing listgmx-users@gromacs.org
> http://lists.gromacs.org/mailman/listinfo/gmx-users
> Please search the archive at http://www.gromacs.org/search before posting!
> Please don't post (un)subscribe requests to the list. Use the www interface
> or send it to gmx-users-requ...@gromacs.org.
> Can't post? Read http://www.gromacs.org/mailing_lists/users.php
>
___
gmx-users mailing listgmx-users@gromacs.org
http://lists.gromacs.org/mailman/listinfo/gmx-users
Please search the archive at http://www.gromacs.org/search before posting!
Please don't post (un)subscribe requests to the list. Use the 
www interface or send it to gmx-users-requ...@gromacs.org.
Can't post? Read http://www.gromacs.org/mailing_lists/users.php

[gmx-users] Load imbalance and Y size error

2009-10-07 Thread nikhil damle
Hi all

 I am trying to simulate a system of 3168 atoms in vacuum in PBC box 
(-d=0.9 nm). Since there are no solvent molecules, i am unable to neutralise 
the system using addition of ions (Hence system has +3 charge). As soon as i 
start the MD run after putting the whole system in box, there is imbalanced 
force of 80% due to which right in the beginning, dynamic load balancing is 
turned on. But even after this, there is ~22% loss of performance and after 
certain steps MD crashes saying :-

"Fatal error: The Y-size of the box (4.618413) times the triclinic skew factor 
(0.866051) is smaller than the number of DD cells (2) times the smallest 
allowed cell size(2.00)"

Can anyone explain the origin of this error and how to deal with it ?

Regards,
Nikhil



  From cricket scores to your friends. Try the Yahoo! India Homepage! 
http://in.yahoo.com/trynew___
gmx-users mailing listgmx-users@gromacs.org
http://lists.gromacs.org/mailman/listinfo/gmx-users
Please search the archive at http://www.gromacs.org/search before posting!
Please don't post (un)subscribe requests to the list. Use the 
www interface or send it to gmx-users-requ...@gromacs.org.
Can't post? Read http://www.gromacs.org/mailing_lists/users.php

Re: [gmx-users] Optimizing a parallel simulation

2009-10-07 Thread Mark Abraham

vivek sharma wrote:

Hi There,
While running a parallel MD simulation, I got following message while 
playing with parameters:

NOTE 3 [file aminoacids.dat, line 1]:
  The optimal PME mesh load for parallel simulations is below 0.5
  and for highly parallel simulations between 0.25 and 0.33,
  for higher performance, increase the cut-off and the PME grid spacing
---

I don't have any idea when a run should be called a highly parallel 
simulation?


Well below 224 :-)


Is it depend on problem-size or the number of processor we are using?


Processors.

I am trying to optimize my simulation to run on 224 processors and 
system consist of:


Total atom count = 54640
where water molecules = 17074


You will want to tweak rcoulomb and fourier_space like the above message 
says. Even so, with less than 500 atoms per particle-particle processor 
you may find yourself spending more time communicating than processing. 
I suspect your system may be too small to get value from so many 
processors, even if you have good network (e.g. Infiniband). You will 
wish to read 3.17 of the manual to start understand the issues involved 
here.



Also, I am getting following message while running grompp
-
NOTE 1 [file md.mdp, line unknown]:
  The Berendsen thermostat does not generate the correct kinetic energy
  distribution. You might want to consider using the V-rescale thermostat.

Any insight in the above message will be highly appreciated.


Seems self-explanatory to me - you're using Berendsen and that might not 
be the right choice. Read the manual sections about the different 
thermostats.


Mark
___
gmx-users mailing listgmx-users@gromacs.org
http://lists.gromacs.org/mailman/listinfo/gmx-users
Please search the archive at http://www.gromacs.org/search before posting!
Please don't post (un)subscribe requests to the list. Use the 
www interface or send it to gmx-users-requ...@gromacs.org.

Can't post? Read http://www.gromacs.org/mailing_lists/users.php


Re: [gmx-users] g_current

2009-10-07 Thread Florian Dommert

Hello Andrew,

 g_current was created in order to calculate the current autocorrelation
 function, conductivity and dielectric constant of an ionic system. Our
 group has written an article describing the methods applied in this
 tool ( J. Chem. Phys. 129(22)(224501), 2008 ), so perhaps a look into
 it will help. In case further questions arise just post them here and
 you will get a reply ASAP.

Cheers,

Flo

* Andrew Paluch  [2009-10-07 21:20:31 -0400]:

Does anyone know of a reference (besides the manual) for the 
calculations performed by g_current?


Thank you,

Andrew
___
gmx-users mailing listgmx-users@gromacs.org
http://lists.gromacs.org/mailman/listinfo/gmx-users
Please search the archive at http://www.gromacs.org/search before posting!
Please don't post (un)subscribe requests to the list. Use thewww 
interface or send it to gmx-users-requ...@gromacs.org.

Can't post? Read http://www.gromacs.org/mailing_lists/users.php


--
Florian Dommert
Dipl.-Phys.

Institute for Computational Physics
University Stuttgart

Pfaffenwaldring 27
70569 Stuttgart

Tel: +49 - 711 / 6856-3613
Fax: +49 - 711 / 6856-3658

EMail: domm...@icp.uni-stuttgart.de
Home: http://www.icp.uni-stuttgart.de/~icp/Florian_Dommert

!! PGP-ENCODED emails preferred !!


signature.asc
Description: Digital signature
___
gmx-users mailing listgmx-users@gromacs.org
http://lists.gromacs.org/mailman/listinfo/gmx-users
Please search the archive at http://www.gromacs.org/search before posting!
Please don't post (un)subscribe requests to the list. Use the 
www interface or send it to gmx-users-requ...@gromacs.org.
Can't post? Read http://www.gromacs.org/mailing_lists/users.php

[gmx-users] Optimizing a parallel simulation

2009-10-07 Thread vivek sharma
Hi There,
While running a parallel MD simulation, I got following message while
playing with parameters:
NOTE 3 [file aminoacids.dat, line 1]:
  The optimal PME mesh load for parallel simulations is below 0.5
  and for highly parallel simulations between 0.25 and 0.33,
  for higher performance, increase the cut-off and the PME grid spacing
---

I don't have any idea when a run should be called a highly parallel
simulation?
Is it depend on problem-size or the number of processor we are using?

I am trying to optimize my simulation to run on 224 processors and system
consist of:

Total atom count = 54640
where water molecules = 17074

Also, I am getting following message while running grompp
-
NOTE 1 [file md.mdp, line unknown]:
  The Berendsen thermostat does not generate the correct kinetic energy
  distribution. You might want to consider using the V-rescale thermostat.

Any insight in the above message will be highly appreciated.

Please advice me how to optimize the parameters to run the simulation with
maximum efficiency.

With thanks in advance,
Vivek
___
gmx-users mailing listgmx-users@gromacs.org
http://lists.gromacs.org/mailman/listinfo/gmx-users
Please search the archive at http://www.gromacs.org/search before posting!
Please don't post (un)subscribe requests to the list. Use the 
www interface or send it to gmx-users-requ...@gromacs.org.
Can't post? Read http://www.gromacs.org/mailing_lists/users.php

RE: [gmx-users] PRODRG

2009-10-07 Thread Smith, Chanel Chonda
In the drug-enzyme tutorial it says that the crude was refined using a
certain force field, SD, and CG. How was this accomplished?

-Original Message-
From: gmx-users-boun...@gromacs.org on behalf of Justin A. Lemkul
Sent: Wed 10/7/2009 1:11 PM
To: Discussion list for GROMACS users
Subject: Re: [gmx-users] PRODRG
 


jorge_quint...@ciencias.uis.edu.co wrote:
> Hello Chanel
> 
> Could you send a copy of the PDB file.  I think that the error is related
> with label atoms included in each force fiel parameter.
> 

More likely this is yet another case of a common misconception about how to
use 
Gromacs.  Specifically, the first error message located under the pdb2gmx 
heading here:

http://www.gromacs.org/Documentation/Errors

-Justin

> See you.
> 
>> Hello,
>>  I have recently made a pdb file using the Dundee PRODRG server.
>> However, when I try to use this pdb in gromacs, I receive an error message
>> that states: "DRG is not in the topology database."  I have tried to use
>> the
>> available tutorial to solve this issue, but with not much success.  Could
>> anyone give me a step by step procedure so that I can use the pdb I have
>> made
>> using PRODRG?
>>
>> Thanks,
>> Chanel King
>> ___
>> gmx-users mailing listgmx-users@gromacs.org
>> http://lists.gromacs.org/mailman/listinfo/gmx-users
>> Please search the archive at http://www.gromacs.org/search before posting!
>> Please don't post (un)subscribe requests to the list. Use the
>> www interface or send it to gmx-users-requ...@gromacs.org.
>> Can't post? Read http://www.gromacs.org/mailing_lists/users.php
>>
> 
> 
> ___
> gmx-users mailing listgmx-users@gromacs.org
> http://lists.gromacs.org/mailman/listinfo/gmx-users
> Please search the archive at http://www.gromacs.org/search before posting!
> Please don't post (un)subscribe requests to the list. Use the 
> www interface or send it to gmx-users-requ...@gromacs.org.
> Can't post? Read http://www.gromacs.org/mailing_lists/users.php
> 

-- 


Justin A. Lemkul
Ph.D. Candidate
ICTAS Doctoral Scholar
Department of Biochemistry
Virginia Tech
Blacksburg, VA
jalemkul[at]vt.edu | (540) 231-9080
http://www.bevanlab.biochem.vt.edu/Pages/Personal/justin


___
gmx-users mailing listgmx-users@gromacs.org
http://lists.gromacs.org/mailman/listinfo/gmx-users
Please search the archive at http://www.gromacs.org/search before posting!
Please don't post (un)subscribe requests to the list. Use the 
www interface or send it to gmx-users-requ...@gromacs.org.
Can't post? Read http://www.gromacs.org/mailing_lists/users.php

<>___
gmx-users mailing listgmx-users@gromacs.org
http://lists.gromacs.org/mailman/listinfo/gmx-users
Please search the archive at http://www.gromacs.org/search before posting!
Please don't post (un)subscribe requests to the list. Use the 
www interface or send it to gmx-users-requ...@gromacs.org.
Can't post? Read http://www.gromacs.org/mailing_lists/users.php

[gmx-users] How does x2top clean dihedrals?

2009-10-07 Thread Hu Zhongqiao
Dear all,

 

Sorry that I have to post this message similar to one posted several
days ago, for I have spent much time on thinking about it but still have
not got a credible explanation.

 

I use x2top to get topology of a capped carbon nanotube with 168 atoms
using oplsaa force field. The output information about dihedreals shows
as follows.

 

Before cleaning: 1008 dihedrals

There are 252 Ryckaert-Bellemans dihedrals, 0 impropers

 

That means one-fourth of initial 1008 dihedrals are kept after cleaning.
After careful check, I find the rule how x2top cleans dihedrals.

 

For example, if I have local structure:
 
C1   C5
\ /
 C3 - C4
/ \
C2   C6

 

Initially four dihedrals are counted by x2top: C1-C3-C4-C5, C1-C3-C4-C6,
C2-C3-C4-C5, and C2-C3-C4-C6.  But after cleaning, three of them are
removed. But I don't clearly understand why three of them should be
removed. Did I lose anything or make a stupid mistake? Sorry that I am
not familiar with C code and so can not get answer from source code. Can
anyone give me some explanation? 

 

Thanks in advance,

 

Sincerely yours,

 

Zhongqiao Hu

 

Dept of Chem & Biomol Engineer

National Univ of Singapore

 

___
gmx-users mailing listgmx-users@gromacs.org
http://lists.gromacs.org/mailman/listinfo/gmx-users
Please search the archive at http://www.gromacs.org/search before posting!
Please don't post (un)subscribe requests to the list. Use the 
www interface or send it to gmx-users-requ...@gromacs.org.
Can't post? Read http://www.gromacs.org/mailing_lists/users.php

Re: [gmx-users] Snapshots in different files

2009-10-07 Thread Arik Cohen
Thanks for replaying. Indeed thats what i did with the BALL library 
which was inefficient. This led me eventually to change the library by 
adding a feature to it.


Any way thanks again

Arik

Dallas B. Warren wrote:


If the time frame between snapshots is not too short, you could have 
that as the length of each MD run, then simple extend the run to keep 
on going.  Could be a bit inefficient.  (This is how you should run MD 
anyway, to ensure don't lose a lot of information if the computer 
system you are running on crashes etc.)


 

I wouldn't be too surprised if it was possible to hack the code so 
that a coordinate file is spat out at some interval (even with a 
limited index of atoms).  But that is something for those that know 
the ins and outs of the code to tell you.


 


Catch ya,

Dr. Dallas Warren
Department of Pharmaceutical Biology
Pharmacy and Pharmaceutical Sciences, Monash University
381 Royal Parade, Parkville VIC 3010
dallas.war...@pharm.monash.edu.au
+61 3 9903 9167
-
When the only tool you own is a hammer, every problem begins to 
resemble a nail.


 

*From:* gmx-users-boun...@gromacs.org 
[mailto:gmx-users-boun...@gromacs.org] *On Behalf Of *Arik Cohen

*Sent:* Thursday, 8 October 2009 9:27 AM
*To:* Discussion list for GROMACS users
*Subject:* Re: [gmx-users] Snapshots in different files

 


Thanks for answering.
This would not be the case so much since another program(sniffer) can 
be working along side gromacs
examining each snapshot(Max 400 residues == atoms. I'm only interested 
in the C-alpha) and then  if all criteria are met to extract/or save 
the coor and if not to erase the snapshot.
The aim here is to do an MD from which an ensemble of the C-alpha will 
be created.


Thanks again

Arik

 
 
Mark Abraham wrote:


Arik Cohen wrote:

Thanks allot, but isn't trjconv should be executed  after the 
trajectory has finished ?. I would like to put each snapshot in a 
different file on the fly.



As Justin said, you can't do that. For starters, it consumes vast 
amounts of disk. Also, it doesn't take long to do it after the fact on 
some workstation, and it is wasteful to spend your (limited) main 
compute resources doing I/O while post-processing output.


GROMACS workflows are intended to run the simulation fast and 
efficiently, and then allow you to process the results with the 
various tools/filters to extract the data you need. You can even 
post-process with mdrun -rerun if you want to get only a subset of 
forces or something. The main exception to this principle is the use 
of xtc-groups, IIRC.


Why do you even want separate PDB frames? Visualization tools like VMD 
will read the trajectory files.


Mark
___
gmx-users mailing listgmx-users@gromacs.org 


http://lists.gromacs.org/mailman/listinfo/gmx-users
Please search the archive at http://www.gromacs.org/search before 
posting!
Please don't post (un)subscribe requests to the list. Use the www 
interface or send it to gmx-users-requ...@gromacs.org 
.

Can't post? Read http://www.gromacs.org/mailing_lists/users.php

 




  
 
 
No virus found in this incoming message.
Checked by AVG - www.avg.com  
Version: 8.5.421 / Virus Database: 270.14.5/2419 - Release Date: 10/07/09 05:18:00
 
  



___
gmx-users mailing listgmx-users@gromacs.org
http://lists.gromacs.org/mailman/listinfo/gmx-users
Please search the archive at http://www.gromacs.org/search before posting!
Please don't post (un)subscribe requests to the list. Use the 
www interface or send it to gmx-users-requ...@gromacs.org.

Can't post? Read http://www.gromacs.org/mailing_lists/users.php



No virus found in this incoming message.
Checked by AVG - www.avg.com 
Version: 8.5.421 / Virus Database: 270.14.5/2419 - Release Date: 10/07/09 05:18:00


  
___
gmx-users mailing listgmx-users@gromacs.org
http://lists.gromacs.org/mailman/listinfo/gmx-users
Please search the archive at http://www.gromacs.org/search before posting!
Please don't post (un)subscribe requests to the list. Use the 
www interface or send it to gmx-users-requ...@gromacs.org.
Can't post? Read http://www.gromacs.org/mailing_lists/users.php

[gmx-users] g_current

2009-10-07 Thread Andrew Paluch
Does anyone know of a reference (besides the manual) for the  
calculations performed by g_current?


Thank you,

Andrew
___
gmx-users mailing listgmx-users@gromacs.org
http://lists.gromacs.org/mailman/listinfo/gmx-users
Please search the archive at http://www.gromacs.org/search before posting!
Please don't post (un)subscribe requests to the list. Use the 
www interface or send it to gmx-users-requ...@gromacs.org.

Can't post? Read http://www.gromacs.org/mailing_lists/users.php


[gmx-users] user defined potential function for 1-4 interaction

2009-10-07 Thread M Hafizur Rahman

Hi All:
I have two separate problems for this subject.
1) What should be the way to pass a formatted table for the 1-4 interaction 
for two different groups? Reading  page 150  and 180 of manual 4.0, I 
understand user defined potential functions can be passed for many groups 
using  energygrp_table in .mdp file but nothing is clearly mentioned 
regarding pair interaction


I have two grps for 1-4 interactions MTN and SFT.
I would write -tablep pair.xvg in mdrun command and pass three tables in the 
name pair.xvg,  pair_MTN_MTN.xvg and  pair_MTN_SFT.xvg.
I recognized the later two talbels are not recognized. Then, what should be 
the correct way.


2) If the same user defined potential function (in a table format) for 1-4 
interaction and non-bonded interaction can be used how the potential 
fuanction can be scaled down for 1-4 interaction. I tried changing 
parameters in [ pairtypes ] but it appears, I got similar distribution and 
shape for 0, .5 and 1 scale.


Any kind of help will be greatly appreciated.

Thanks,
M. H. Rahman

___
gmx-users mailing listgmx-users@gromacs.org
http://lists.gromacs.org/mailman/listinfo/gmx-users
Please search the archive at http://www.gromacs.org/search before posting!
Please don't post (un)subscribe requests to the list. Use the 
www interface or send it to gmx-users-requ...@gromacs.org.

Can't post? Read http://www.gromacs.org/mailing_lists/users.php


[gmx-users] g_dipole for spce

2009-10-07 Thread Jamie Seyed
Dear all,
I tried to use g_dipole for a SPC/E water system and I got these results.
-
Dipole moment (Debye)
-
Average  =   2.3506  Std. Dev. =   0.0143  Error =   0.0001
The following averages for the complete trajectory have been calculated:
 Total < M_x > = 4.34454 Debye
 Total < M_y > = -10.3785 Debye
 Total < M_z > = -2.2976 Debye
 Total < M_x^2 > = 239.768 Debye^2
 Total < M_y^2 > = 557.705 Debye^2
 Total < M_z^2 > = 541.547 Debye^2
 Total < |M|^2 > = 1339.02 Debye^2
 Total < |M| >^2 = 131.868 Debye^2
 < |M|^2 > - < |M| >^2 = 1207.15 Debye^2
Finite system Kirkwood g factor G_k = 1.01149
Infinite system Kirkwood g factor g_k = 0.691347
Epsilon = 19.810
--
Are they reasonable results (for example Epsilon = 19.810) and others?? I
checked the average value  2.3506 with literatures and it seems ok. I
appreciate if someone have a look at them and let me know if I have any
mistake or not. Many Thanks in Advance/Jamie
___
gmx-users mailing listgmx-users@gromacs.org
http://lists.gromacs.org/mailman/listinfo/gmx-users
Please search the archive at http://www.gromacs.org/search before posting!
Please don't post (un)subscribe requests to the list. Use the 
www interface or send it to gmx-users-requ...@gromacs.org.
Can't post? Read http://www.gromacs.org/mailing_lists/users.php

RE: [gmx-users] Snapshots in different files

2009-10-07 Thread Dallas B. Warren
If the time frame between snapshots is not too short, you could have
that as the length of each MD run, then simple extend the run to keep on
going.  Could be a bit inefficient.  (This is how you should run MD
anyway, to ensure don't lose a lot of information if the computer system
you are running on crashes etc.)

 

I wouldn't be too surprised if it was possible to hack the code so that
a coordinate file is spat out at some interval (even with a limited
index of atoms).  But that is something for those that know the ins and
outs of the code to tell you.

 

Catch ya,

Dr. Dallas Warren
Department of Pharmaceutical Biology 
Pharmacy and Pharmaceutical Sciences, Monash University
381 Royal Parade, Parkville VIC 3010
dallas.war...@pharm.monash.edu.au
+61 3 9903 9167
-
When the only tool you own is a hammer, every problem begins to resemble
a nail. 

 

From: gmx-users-boun...@gromacs.org
[mailto:gmx-users-boun...@gromacs.org] On Behalf Of Arik Cohen
Sent: Thursday, 8 October 2009 9:27 AM
To: Discussion list for GROMACS users
Subject: Re: [gmx-users] Snapshots in different files

 

Thanks for answering. 
This would not be the case so much since another program(sniffer) can be
working along side gromacs
examining each snapshot(Max 400 residues == atoms. I'm only interested
in the C-alpha) and then  if all criteria are met to extract/or save the
coor and if not to erase the snapshot.
The aim here is to do an MD from which an ensemble of the C-alpha will
be created.

Thanks again 

Arik

 
 
Mark Abraham wrote: 

Arik Cohen wrote: 



Thanks allot, but isn't trjconv should be executed  after the trajectory
has finished ?. I would like to put each snapshot in a different file on
the fly. 


As Justin said, you can't do that. For starters, it consumes vast
amounts of disk. Also, it doesn't take long to do it after the fact on
some workstation, and it is wasteful to spend your (limited) main
compute resources doing I/O while post-processing output. 

GROMACS workflows are intended to run the simulation fast and
efficiently, and then allow you to process the results with the various
tools/filters to extract the data you need. You can even post-process
with mdrun -rerun if you want to get only a subset of forces or
something. The main exception to this principle is the use of
xtc-groups, IIRC. 

Why do you even want separate PDB frames? Visualization tools like VMD
will read the trajectory files. 

Mark 
___ 
gmx-users mailing listgmx-users@gromacs.org 
http://lists.gromacs.org/mailman/listinfo/gmx-users 
Please search the archive at http://www.gromacs.org/search before
posting! 
Please don't post (un)subscribe requests to the list. Use the www
interface or send it to gmx-users-requ...@gromacs.org. 
Can't post? Read http://www.gromacs.org/mailing_lists/users.php



 






 
 
No virus found in this incoming message.
Checked by AVG - www.avg.com 
Version: 8.5.421 / Virus Database: 270.14.5/2419 - Release Date:
10/07/09 05:18:00
 
  
___
gmx-users mailing listgmx-users@gromacs.org
http://lists.gromacs.org/mailman/listinfo/gmx-users
Please search the archive at http://www.gromacs.org/search before posting!
Please don't post (un)subscribe requests to the list. Use the 
www interface or send it to gmx-users-requ...@gromacs.org.
Can't post? Read http://www.gromacs.org/mailing_lists/users.php

Re: [gmx-users] Problem in equilibration for membrane system

2009-10-07 Thread Itamar Kass
hi Bing,

I advice you to look carefully on any clashes between POPC and the
protein. It might happen that one lipid was 'trapped' inside the
protein or more likely that atom water had trapped and that what crash
you system.

Cheers,
Itamar

---

 "In theory, there is no difference between theory and practice. But,
in practice, there is." - Jan L.A. van de Snepscheut

===
| Itamar Kass, Ph.D.
| Postdoctoral Research Fellow
|
| Department of Biochemistry and Molecular Biology
| Building 77 Clayton Campus
| Wellington Road
| Monash University,
| Victoria 3800
| Australia
|
| Tel: +61 3 9902 9376
| Fax: +61 3 9902 9500
| E-mail: itamar.k...@med.monash.edu.au


On Wed, Oct 7, 2009 at 9:47 PM, Justin A. Lemkul  wrote:
>
>
> Bing Bing wrote:
>>
>> My protein complex was minimized initially before putting into the lipid.
>> POPC is also a preequilibrated structure from Tielemen website. I don't
>> quite understand on what you meant by "the protein is not happy with the
>> restraint". The system went well through out the nvt, npt with
>
> I think what Mark was getting as was if you're restraining some unusual or
> inappropriate geometry, you can get nasty results when you turn off
> restraints.  I don't know that this is the case in your situation.
>
>> restraint on protein Once it move to md production run ( which is
>> without restraint on protein), error occurred (range checking error). The
>> reason i'm using the version3.3.2 is because the forcefield i'm using at the
>> moment only support version3.3.2.  Is my problem cause by
>
> I don't understand this statement.  The force field you mentioned is most
> certainly supported in the newest version of Gromacs, in fact my tutorial
> was written for the version 4.0.x series.  Please also see the advanced
> troubleshooting page I provide; you may find some ideas there.
>
> Certainly, watch the trajectory and see where things are going wrong to
> deduce the nature of the issue.
>
> -Justin
>
>> bug of the older version? is there any bugfix fot this? please
>> help.
>>
>> regards,
>> Bing
>>
>>
>> On Wed, Oct 7, 2009 at 4:15 PM, Mark Abraham > > wrote:
>>
>>    Bing Bing wrote:
>>
>>        Dear All,
>>        I'm running the protein complex with POPC system. NVT (100ps)
>>        and NPT (200ps) were done with restraint on the protein follow
>>        by 1000 ps of NPT without restraint on protein. The trajectory
>>        was checked in term of pressure, potential, area per lipid
>>        were checked and everything seems ok (potential almost
>>        plateu, area per lipid close to experimental value). I proceeded
>>        with md (similar to Justin's tutorial) for 1 ns. The simulation
>>        stop around 800ps with the range checking error.
>>
>>
>>    OK so look at the collective variables, and/or trajectory to find
>>    evidence of the problem. The most likely explanation is that your
>>    protein was unhappy in the restraints, of course. Perhaps you should
>>    try some vacuum EM before you embed it in the lipid, or some such.
>>
>>
>>        I'm using gromacs 3.3.2, with GROMOS9653a6 forcefield. I tired
>>        to be more gentle while releasing the restraint.Different
>>        stepwise of releasing the restraint during NPT were tried and
>>        followed by production MD, all the simulation stop or i shall
>>        said it crashed around ~800ps with the same error (range
>>        checking error). I'm out of idea what is the problem here.
>>        Please advice.
>>
>>
>>    Range checking is just one of many things that can be the first
>>    thing to break when there's an underlying systemic problem.
>>
>>    Also, unless you need scientific continuity, 3.3.2 is about 4 years
>>    old... 4.0.5 will be much faster!
>>
>>    Mark
>>    ___
>>    gmx-users mailing list    gmx-us...@gromacs.org
>>    
>>    http://lists.gromacs.org/mailman/listinfo/gmx-users
>>    Please search the archive at http://www.gromacs.org/search before
>>    posting!
>>    Please don't post (un)subscribe requests to the list. Use the www
>>    interface or send it to gmx-users-requ...@gromacs.org
>>    .
>>    Can't post? Read http://www.gromacs.org/mailing_lists/users.php
>>
>>
>>
>> 
>>
>> ___
>> gmx-users mailing list    gmx-us...@gromacs.org
>> http://lists.gromacs.org/mailman/listinfo/gmx-users
>> Please search the archive at http://www.gromacs.org/search before posting!
>> Please don't post (un)subscribe requests to the list. Use the www
>> interface or send it to gmx-users-requ...@gromacs.org.
>> Can't post? Read http://www.gromacs.org/mailing_lists/users.php
>
> --
> 
>
> Just

Re: [gmx-users] Distance restraint problem with 4.0.5 version

2009-10-07 Thread Mark Abraham

Alexandre Suman de Araujo wrote:

Hi Gmxers

Some months ago I performed some simulations, with GMX 3.3.3 version, 
where I apply distance restraint between water oxygen atoms and a static 
dummy site located at the center of a sphere to keep waters inside this 
sphere. After performed an upgrade to GMX 4.0.5 version this distance 
restraint scheme stopped to work.
I read distance restraint material in 4.0.5 paper manual and I can't 
find any modification reported there.


Looking at gmxdump output from tpr's generated with grompp from 3.3.3 
and 4.0.5 versions and the same .mdp, .gro and .top files, I had this:


3.3.3 version: grep -i disre gmxdump3.txt |more

  disre= Simple
  disre_weighting  = Conservative
  disre_mixed  = FALSE
  nstdisreout  = 100
functype[468]=DISRES, label=   0, type=1, low= 0.e+00, 
up1= 3.e+00, up2= 9.3962e+00, fac= 1.e+00)
functype[469]=DISRES, label=   1, type=1, low= 0.e+00, 
up1= 3.e+00, up2= 9.3962e+00, fac= 1.e+00)
functype[470]=DISRES, label=   2, type=1, low= 0.e+00, 
up1= 3.e+00, up2= 9.3962e+00, fac= 1.e+00)
functype[471]=DISRES, label=   3, type=1, low= 0.e+00, 
up1= 3.e+00, up2= 9.3962e+00, fac= 1.e+00)
functype[472]=DISRES, label=   4, type=1, low= 0.e+00, 
up1= 3.e+00, up2= 9.3962e+00, fac= 1.e+00)
functype[473]=DISRES, label=   5, type=1, low= 0.e+00, 
up1= 3.e+00, up2= 9.3962e+00, fac= 1.e+00)

.
.
.
functype[724]=DISRES, label= 256, type=1, low= 0.e+00, 
up1= 3.e+00, up2= 9.3962e+00, fac= 1.e+00)
functype[725]=DISRES, label= 257, type=1, low= 0.e+00, 
up1= 3.e+00, up2= 9.3962e+00, fac= 1.e+00)
functype[726]=DISRES, label= 258, type=1, low= 0.e+00, 
up1= 3.e+00, up2= 9.3962e+00, fac= 1.e+00)

   0 type=468 (DISRES) 389 392
   1 type=469 (DISRES) 393 396
   2 type=470 (DISRES) 397 400
   3 type=471 (DISRES) 401 404
   4 type=472 (DISRES) 405 408
   5 type=473 (DISRES) 409 412
   .
   .
   .
   256 type=724 (DISRES) 1413 1416
   257 type=725 (DISRES) 1417 1420
   258 type=726 (DISRES) 1421 1424

4.0.5 version: grep -i disre gmxdump4.txt

  disre= Simple
  disre_weighting  = Conservative
  disre_mixed  = FALSE
  nstdisreout  = 100
functype[470]=DISRES, label=   0, type=1, low= 0.e+00, 
up1= 3.e+00, up2= 9.3962e+00, fac= 1.e+00)

   0 type=470 (DISRES) 0 3


It is clear that 3.3.3 version is applying the constraints to all waters 
(what is correct) and 4.0.5 to just one. However, I can't find a way to 
fix this.


Hmm, the above indicates that 3.3.3 wasn't folding the function type 
definitions correctly, and apparently 4.0.5 is folding the types as well 
as the references to them. If you're sure you've used the same 
.mdp/.top, then please submit a bugzilla (http://bugzilla.gromacs.org/) 
and attach a .tar of the .top, .mdp, .tpr and .gro files.


Also have a look at grep -i disre -C 5 gmxdump4.txt to see if there's 
anything interesting in the local contexts of the differences.


Mark

Does anyone could help me with this issue? Are there modifications in 
distance restraint algorithm in 4.0.5 version compared with 3.3.3 
version? Or this can be a bug?


Thanks


___
gmx-users mailing listgmx-users@gromacs.org
http://lists.gromacs.org/mailman/listinfo/gmx-users
Please search the archive at http://www.gromacs.org/search before posting!
Please don't post (un)subscribe requests to the list. Use the 
www interface or send it to gmx-users-requ...@gromacs.org.

Can't post? Read http://www.gromacs.org/mailing_lists/users.php


Re: [gmx-users] trjconv to process multiple types of molecules

2009-10-07 Thread Mark Abraham

Yan Gao wrote:

Hi There,

I have a problem using trjconv to get the trajectory.
I have three types of molecules: MOL, ION, SOL
and I want to convert only 2 types of them, how ever trjconv seems only 
accept one input.
I want to keep only MOL and ION, and avoid the large amount of SOL, 
which will reduce the analysis a lot.


Any suggestions will be appreciated!

Group 0 (  System) has 14260 elements
Group 1 ( MOL) has  2640 elements
Group 2 ( SOL) has 11580 elements
Group 3 ( ION) has40 elements


Use make_ndx to create a non-SOL group, and supply the resulting index 
file to trjconv.


Mark
___
gmx-users mailing listgmx-users@gromacs.org
http://lists.gromacs.org/mailman/listinfo/gmx-users
Please search the archive at http://www.gromacs.org/search before posting!
Please don't post (un)subscribe requests to the list. Use the 
www interface or send it to gmx-users-requ...@gromacs.org.

Can't post? Read http://www.gromacs.org/mailing_lists/users.php


Re: [gmx-users] trjconv to process multiple types of molecules

2009-10-07 Thread Justin A. Lemkul



Yan Gao wrote:

Hi There,

I have a problem using trjconv to get the trajectory.
I have three types of molecules: MOL, ION, SOL
and I want to convert only 2 types of them, how ever trjconv seems only 
accept one input.
I want to keep only MOL and ION, and avoid the large amount of SOL, 
which will reduce the analysis a lot.


Any suggestions will be appreciated!



Create an index group that contains the species you want.

-Justin


Group 0 (  System) has 14260 elements
Group 1 ( MOL) has  2640 elements
Group 2 ( SOL) has 11580 elements
Group 3 ( ION) has40 elements

Warm regards,
Stone
___
gmx-users mailing listgmx-users@gromacs.org
http://lists.gromacs.org/mailman/listinfo/gmx-users
Please search the archive at http://www.gromacs.org/search before posting!
Please don't post (un)subscribe requests to the list. Use the www 
interface or send it to gmx-users-requ...@gromacs.org.

Can't post? Read http://www.gromacs.org/mailing_lists/users.php



--


Justin A. Lemkul
Ph.D. Candidate
ICTAS Doctoral Scholar
Department of Biochemistry
Virginia Tech
Blacksburg, VA
jalemkul[at]vt.edu | (540) 231-9080
http://www.bevanlab.biochem.vt.edu/Pages/Personal/justin


___
gmx-users mailing listgmx-users@gromacs.org
http://lists.gromacs.org/mailman/listinfo/gmx-users
Please search the archive at http://www.gromacs.org/search before posting!
Please don't post (un)subscribe requests to the list. Use the 
www interface or send it to gmx-users-requ...@gromacs.org.

Can't post? Read http://www.gromacs.org/mailing_lists/users.php


Re: [gmx-users] trjconv to process multiple types of molecules

2009-10-07 Thread TJ Piggot

Hi,

You need to use make_ndx to merge the MOL and ION into a new index group 
and then use that group in trjconv


Cheers

Tom

--On 07 October 2009 15:32 -0700 Yan Gao  wrote:


Hi There,

I have a problem using trjconv to get the trajectory.
I have three types of molecules: MOL, ION, SOL
and I want to convert only 2 types of them, how ever trjconv seems only
accept one input.
I want to keep only MOL and ION, and avoid the large amount of SOL, which
will reduce the analysis a lot.

Any suggestions will be appreciated!

Group 0 (  System) has 14260 elements
Group 1 ( MOL) has  2640 elements
Group 2 ( SOL) has 11580 elements
Group 3 ( ION) has40 elements

Warm regards,
Stone
___
gmx-users mailing listgmx-users@gromacs.org
http://lists.gromacs.org/mailman/listinfo/gmx-users
Please search the archive at http://www.gromacs.org/search before posting!
Please don't post (un)subscribe requests to the list. Use the www
interface or send it to gmx-users-requ...@gromacs.org.
Can't post? Read http://www.gromacs.org/mailing_lists/users.php




--
TJ Piggot
t.pig...@bristol.ac.uk
University of Bristol, UK.

___
gmx-users mailing listgmx-users@gromacs.org
http://lists.gromacs.org/mailman/listinfo/gmx-users
Please search the archive at http://www.gromacs.org/search before posting!
Please don't post (un)subscribe requests to the list. Use the 
www interface or send it to gmx-users-requ...@gromacs.org.

Can't post? Read http://www.gromacs.org/mailing_lists/users.php


[gmx-users] trjconv to process multiple types of molecules

2009-10-07 Thread Yan Gao

Hi There,

I have a problem using trjconv to get the trajectory.
I have three types of molecules: MOL, ION, SOL
and I want to convert only 2 types of them, how ever trjconv seems only 
accept one input.
I want to keep only MOL and ION, and avoid the large amount of SOL, which 
will reduce the analysis a lot.


Any suggestions will be appreciated!

Group 0 (  System) has 14260 elements
Group 1 ( MOL) has  2640 elements
Group 2 ( SOL) has 11580 elements
Group 3 ( ION) has40 elements

Warm regards,
Stone 


___
gmx-users mailing listgmx-users@gromacs.org
http://lists.gromacs.org/mailman/listinfo/gmx-users
Please search the archive at http://www.gromacs.org/search before posting!
Please don't post (un)subscribe requests to the list. Use the 
www interface or send it to gmx-users-requ...@gromacs.org.

Can't post? Read http://www.gromacs.org/mailing_lists/users.php


Re: [gmx-users] Snapshots in different files

2009-10-07 Thread Arik Cohen

Thanks for answering.
This would not be the case so much since another program(sniffer) can be 
working along side gromacs
examining each snapshot(Max 400 residues == atoms. I'm only interested 
in the C-alpha) and then  if all criteria are met to extract/or save the 
coor and if not to erase the snapshot.
The aim here is to do an MD from which an ensemble of the C-alpha will 
be created.


Thanks again

Arik



Mark Abraham wrote:

Arik Cohen wrote:
Thanks allot, but isn't trjconv should be executed  after the 
trajectory has finished ?. I would like to put each snapshot in a 
different file on the fly.


As Justin said, you can't do that. For starters, it consumes vast 
amounts of disk. Also, it doesn't take long to do it after the fact on 
some workstation, and it is wasteful to spend your (limited) main 
compute resources doing I/O while post-processing output.


GROMACS workflows are intended to run the simulation fast and 
efficiently, and then allow you to process the results with the 
various tools/filters to extract the data you need. You can even 
post-process with mdrun -rerun if you want to get only a subset of 
forces or something. The main exception to this principle is the use 
of xtc-groups, IIRC.


Why do you even want separate PDB frames? Visualization tools like VMD 
will read the trajectory files.


Mark
___
gmx-users mailing listgmx-users@gromacs.org
http://lists.gromacs.org/mailman/listinfo/gmx-users
Please search the archive at http://www.gromacs.org/search before 
posting!
Please don't post (un)subscribe requests to the list. Use the www 
interface or send it to gmx-users-requ...@gromacs.org.

Can't post? Read http://www.gromacs.org/mailing_lists/users.php



No virus found in this incoming message.
Checked by AVG - www.avg.com 
Version: 8.5.421 / Virus Database: 270.14.5/2419 - Release Date: 10/07/09 05:18:00


  
___
gmx-users mailing listgmx-users@gromacs.org
http://lists.gromacs.org/mailman/listinfo/gmx-users
Please search the archive at http://www.gromacs.org/search before posting!
Please don't post (un)subscribe requests to the list. Use the 
www interface or send it to gmx-users-requ...@gromacs.org.
Can't post? Read http://www.gromacs.org/mailing_lists/users.php

Re: [gmx-users] Snapshots in different files

2009-10-07 Thread Arik Cohen

Thanks allot for all your help and ultrafast response.

Arik

Justin A. Lemkul wrote:



Arik Cohen wrote:
Thanks allot, but isn't trjconv should be executed  after the 
trajectory has finished ?. I would like to put each snapshot in a 
different file on the fly.




Not possible, as far as I'm aware.

-Justin


Thanks

Arik

Justin A. Lemkul wrote:



Arik Cohen wrote:

Dear users,

Is there a way to take a snapshot along the trajectory and write it 
into a different file, so that each snapshot will be written into 
its own file named with its own index ?(e.g. snap_1, snap_2 )




trjconv -sep

-Justin


Thanks

Arik
___
gmx-users mailing listgmx-users@gromacs.org
http://lists.gromacs.org/mailman/listinfo/gmx-users
Please search the archive at http://www.gromacs.org/search before 
posting!
Please don't post (un)subscribe requests to the list. Use the www 
interface or send it to gmx-users-requ...@gromacs.org.

Can't post? Read http://www.gromacs.org/mailing_lists/users.php



 




No virus found in this incoming message.
Checked by AVG - www.avg.com Version: 8.5.421 / Virus Database: 
270.14.5/2419 - Release Date: 10/07/09 05:18:00


  





No virus found in this incoming message.
Checked by AVG - www.avg.com 
Version: 8.5.421 / Virus Database: 270.14.5/2419 - Release Date: 10/07/09 05:18:00


  
___
gmx-users mailing listgmx-users@gromacs.org
http://lists.gromacs.org/mailman/listinfo/gmx-users
Please search the archive at http://www.gromacs.org/search before posting!
Please don't post (un)subscribe requests to the list. Use the 
www interface or send it to gmx-users-requ...@gromacs.org.
Can't post? Read http://www.gromacs.org/mailing_lists/users.php

Re: [gmx-users] Snapshots in different files

2009-10-07 Thread Mark Abraham

Arik Cohen wrote:
Thanks allot, but isn't trjconv should be executed  after the trajectory 
has finished ?. I would like to put each snapshot in a different file on 
the fly.


As Justin said, you can't do that. For starters, it consumes vast 
amounts of disk. Also, it doesn't take long to do it after the fact on 
some workstation, and it is wasteful to spend your (limited) main 
compute resources doing I/O while post-processing output.


GROMACS workflows are intended to run the simulation fast and 
efficiently, and then allow you to process the results with the various 
tools/filters to extract the data you need. You can even post-process 
with mdrun -rerun if you want to get only a subset of forces or 
something. The main exception to this principle is the use of 
xtc-groups, IIRC.


Why do you even want separate PDB frames? Visualization tools like VMD 
will read the trajectory files.


Mark
___
gmx-users mailing listgmx-users@gromacs.org
http://lists.gromacs.org/mailman/listinfo/gmx-users
Please search the archive at http://www.gromacs.org/search before posting!
Please don't post (un)subscribe requests to the list. Use the 
www interface or send it to gmx-users-requ...@gromacs.org.

Can't post? Read http://www.gromacs.org/mailing_lists/users.php


Re: [gmx-users] Snapshots in different files

2009-10-07 Thread Justin A. Lemkul



Arik Cohen wrote:
Thanks allot, but isn't trjconv should be executed  after the trajectory 
has finished ?. I would like to put each snapshot in a different file on 
the fly.




Not possible, as far as I'm aware.

-Justin


Thanks

Arik

Justin A. Lemkul wrote:



Arik Cohen wrote:

Dear users,

Is there a way to take a snapshot along the trajectory and write it 
into a different file, so that each snapshot will be written into its 
own file named with its own index ?(e.g. snap_1, snap_2 )




trjconv -sep

-Justin


Thanks

Arik
___
gmx-users mailing listgmx-users@gromacs.org
http://lists.gromacs.org/mailman/listinfo/gmx-users
Please search the archive at http://www.gromacs.org/search before 
posting!
Please don't post (un)subscribe requests to the list. Use the www 
interface or send it to gmx-users-requ...@gromacs.org.

Can't post? Read http://www.gromacs.org/mailing_lists/users.php






No virus found in this incoming message.
Checked by AVG - www.avg.com 
Version: 8.5.421 / Virus Database: 270.14.5/2419 - Release Date: 10/07/09 05:18:00


  


--


Justin A. Lemkul
Ph.D. Candidate
ICTAS Doctoral Scholar
Department of Biochemistry
Virginia Tech
Blacksburg, VA
jalemkul[at]vt.edu | (540) 231-9080
http://www.bevanlab.biochem.vt.edu/Pages/Personal/justin


___
gmx-users mailing listgmx-users@gromacs.org
http://lists.gromacs.org/mailman/listinfo/gmx-users
Please search the archive at http://www.gromacs.org/search before posting!
Please don't post (un)subscribe requests to the list. Use the 
www interface or send it to gmx-users-requ...@gromacs.org.

Can't post? Read http://www.gromacs.org/mailing_lists/users.php


Re: [gmx-users] Extended structure for Estrogen receptor

2009-10-07 Thread Mark Abraham

Pradip Biswas wrote:

Dear All,

All the estrogen receptor alpha structure that I found on PDB 
repository, contain mainly the ligand binding domain (residues 301-550). 
I am looking for a structure that has the hinge region (240-300) as 
well. Does anyone have any extended structure (even if it is an 
engineered one) of Estrogen Receptor Alpha which contains the ligand 
binding domain plus the hinge region residues: 240-550 ??   


Ligand binding domains can be disordered, which makes them unsuited for 
X-ray or NMR structure determination. Thus they might be missing from 
the results or indeed excised before structure determination. Read the 
comments in the PDB files and/or literature to see.


Mark
___
gmx-users mailing listgmx-users@gromacs.org
http://lists.gromacs.org/mailman/listinfo/gmx-users
Please search the archive at http://www.gromacs.org/search before posting!
Please don't post (un)subscribe requests to the list. Use the 
www interface or send it to gmx-users-requ...@gromacs.org.

Can't post? Read http://www.gromacs.org/mailing_lists/users.php


Re: [gmx-users] Snapshots in different files

2009-10-07 Thread Arik Cohen
Thanks allot, but isn't trjconv should be executed  after the trajectory 
has finished ?. I would like to put each snapshot in a different file on 
the fly.


Thanks

Arik

Justin A. Lemkul wrote:



Arik Cohen wrote:

Dear users,

Is there a way to take a snapshot along the trajectory and write it 
into a different file, so that each snapshot will be written into its 
own file named with its own index ?(e.g. snap_1, snap_2 )




trjconv -sep

-Justin


Thanks

Arik
___
gmx-users mailing listgmx-users@gromacs.org
http://lists.gromacs.org/mailman/listinfo/gmx-users
Please search the archive at http://www.gromacs.org/search before 
posting!
Please don't post (un)subscribe requests to the list. Use the www 
interface or send it to gmx-users-requ...@gromacs.org.

Can't post? Read http://www.gromacs.org/mailing_lists/users.php






No virus found in this incoming message.
Checked by AVG - www.avg.com 
Version: 8.5.421 / Virus Database: 270.14.5/2419 - Release Date: 10/07/09 05:18:00


  
___
gmx-users mailing listgmx-users@gromacs.org
http://lists.gromacs.org/mailman/listinfo/gmx-users
Please search the archive at http://www.gromacs.org/search before posting!
Please don't post (un)subscribe requests to the list. Use the 
www interface or send it to gmx-users-requ...@gromacs.org.
Can't post? Read http://www.gromacs.org/mailing_lists/users.php

Re: [gmx-users] Snapshots in different files

2009-10-07 Thread Justin A. Lemkul



Arik Cohen wrote:

Dear users,

Is there a way to take a snapshot along the trajectory and write it into 
a different file, so that each snapshot will be written into its own 
file named with its own index ?(e.g. snap_1, snap_2 )




trjconv -sep

-Justin


Thanks

Arik
___
gmx-users mailing listgmx-users@gromacs.org
http://lists.gromacs.org/mailman/listinfo/gmx-users
Please search the archive at http://www.gromacs.org/search before posting!
Please don't post (un)subscribe requests to the list. Use the www 
interface or send it to gmx-users-requ...@gromacs.org.

Can't post? Read http://www.gromacs.org/mailing_lists/users.php



--


Justin A. Lemkul
Ph.D. Candidate
ICTAS Doctoral Scholar
Department of Biochemistry
Virginia Tech
Blacksburg, VA
jalemkul[at]vt.edu | (540) 231-9080
http://www.bevanlab.biochem.vt.edu/Pages/Personal/justin


___
gmx-users mailing listgmx-users@gromacs.org
http://lists.gromacs.org/mailman/listinfo/gmx-users
Please search the archive at http://www.gromacs.org/search before posting!
Please don't post (un)subscribe requests to the list. Use the 
www interface or send it to gmx-users-requ...@gromacs.org.

Can't post? Read http://www.gromacs.org/mailing_lists/users.php


[gmx-users] Snapshots in different files

2009-10-07 Thread Arik Cohen

Dear users,

Is there a way to take a snapshot along the trajectory and write it into 
a different file, so that each snapshot will be written into its own 
file named with its own index ?(e.g. snap_1, snap_2 )


Thanks

Arik
___
gmx-users mailing listgmx-users@gromacs.org
http://lists.gromacs.org/mailman/listinfo/gmx-users
Please search the archive at http://www.gromacs.org/search before posting!
Please don't post (un)subscribe requests to the list. Use the 
www interface or send it to gmx-users-requ...@gromacs.org.

Can't post? Read http://www.gromacs.org/mailing_lists/users.php


Re: [gmx-users] mailing list search

2009-10-07 Thread Rossen Apostolov
Hi,

I made some updates and incorrectly changed the permissions.
You should be able to search now even without logging in.

Rossen

Sun Joo Lee wrote:
> Hello
>
> I have been trying to search the mailing list after I logged in to the
> new gromacs website. 
> But I do not get any searched results but "This page is restricted."
> comment.
>
> Could anyone tell me what the problem might be? 
>
>
> Thank you
> Sunjoo 
> 
>
> ___
> gmx-users mailing listgmx-users@gromacs.org
> http://lists.gromacs.org/mailman/listinfo/gmx-users
> Please search the archive at http://www.gromacs.org/search before posting!
> Please don't post (un)subscribe requests to the list. Use the 
> www interface or send it to gmx-users-requ...@gromacs.org.
> Can't post? Read http://www.gromacs.org/mailing_lists/users.php
___
gmx-users mailing listgmx-users@gromacs.org
http://lists.gromacs.org/mailman/listinfo/gmx-users
Please search the archive at http://www.gromacs.org/search before posting!
Please don't post (un)subscribe requests to the list. Use the 
www interface or send it to gmx-users-requ...@gromacs.org.
Can't post? Read http://www.gromacs.org/mailing_lists/users.php


[gmx-users] Problems with FENE potential

2009-10-07 Thread Marisa Roman
Hi there,
I have been trying to run a simulation using the fene potential option in a 
chain of beads as model for my protein, ubiquitin and the sd integrator.
The problem comes when one of the beads goes beyond the limits and comes out 
from the other side of the box, the distance comes out taking into account the 
position on the other side of the box and not the real one. Therefore my system 
collapses.
Any suggestion on this will be very appreciated.

Marisa Roman
Philadelphia, US

PS: I could print out coordinates of the beads involved to see the problem:

step 970, will finish at Wed Oct  7 13:42:36 2009BONDS: x[42] = (3.533585 
2.817612 14.356295 ); x[43] = (3.469475 2.807785 14.618975 )  dr2= 0.0732075 
BONDS: x[42] = (3.533310 2.820071 14.356031 ); x[43] = (3.467908 2.807405 
14.620831 )  dr2= 0.0745564 
BONDS: x[42] = (3.532996 2.822756 14.356280 ); x[43] = (3.466114 2.806527 
14.622769 )  dr2= 0.0757529 
BONDS: x[42] = (3.532656 2.825698 14.357033 ); x[43] = (3.464116 2.805169 
14.624761 )  dr2= 0.0767974 
BONDS: x[42] = (3.532295 2.828872 14.358303 ); x[43] = (3.461922 2.803345 
14.626786 )  dr2= 0.0776872 
BONDS: x[42] = (3.531900 2.832249 14.360102 ); x[43] = (3.459582 2.801082 
14.628803 )  dr2= 0.0784018 
BONDS: x[42] = (3.531463 2.835809 14.362425 ); x[43] = (3.457096 2.798412 
14.630804 )  dr2= 0.0789566 
BONDS: x[42] = (3.530972 2.839528 14.365273 ); x[43] = (3.454516 2.795343 
14.632739 )  dr2= 0.0793356 
BONDS: x[42] = (3.530426 2.843364 14.368648 ); x[43] = (3.451869 2.791898 
14.634576 )  dr2= 0.0795378 
BONDS: x[42] = (3.529795 2.847286 14.372531 ); x[43] = (3.449184 11.288118 
14.636299 )  dr2=   71.3237 

---
Program mdrun, VERSION 4.0.5
Source code file: bondfree.c, line: 325

Fatal error:
 x[42] = (3.529795 2.847286 14.372531 ); x[43] = (3.449184 11.288118 14.636299 
);
 r^2 (71.323738) >= bm^2 (0.324900) in FENE bond between atoms 43 and 44
---

"The Path Of the Righteous Man is Beset On All Sides With the Iniquities Of the 
Selfish and the Tyranny Of Evil Men." (Pulp Fiction)



___
gmx-users mailing listgmx-users@gromacs.org
http://lists.gromacs.org/mailman/listinfo/gmx-users
Please search the archive at http://www.gromacs.org/search before posting!
Please don't post (un)subscribe requests to the list. Use the 
www interface or send it to gmx-users-requ...@gromacs.org.
Can't post? Read http://www.gromacs.org/mailing_lists/users.php


[gmx-users] mailing list search

2009-10-07 Thread Sun Joo Lee

Hello

I have been trying to search the mailing list after I logged in to the  
new gromacs website.
But I do not get any searched results but "This page is restricted."  
comment.


Could anyone tell me what the problem might be?


Thank you
Sunjoo ___
gmx-users mailing listgmx-users@gromacs.org
http://lists.gromacs.org/mailman/listinfo/gmx-users
Please search the archive at http://www.gromacs.org/search before posting!
Please don't post (un)subscribe requests to the list. Use the 
www interface or send it to gmx-users-requ...@gromacs.org.
Can't post? Read http://www.gromacs.org/mailing_lists/users.php

Re: [gmx-users] PRODRG

2009-10-07 Thread Justin A. Lemkul



jorge_quint...@ciencias.uis.edu.co wrote:

Hello Chanel

Could you send a copy of the PDB file.  I think that the error is related
with label atoms included in each force fiel parameter.



More likely this is yet another case of a common misconception about how to use 
Gromacs.  Specifically, the first error message located under the pdb2gmx 
heading here:


http://www.gromacs.org/Documentation/Errors

-Justin


See you.


Hello,
 I have recently made a pdb file using the Dundee PRODRG server.
However, when I try to use this pdb in gromacs, I receive an error message
that states: "DRG is not in the topology database."  I have tried to use
the
available tutorial to solve this issue, but with not much success.  Could
anyone give me a step by step procedure so that I can use the pdb I have
made
using PRODRG?

Thanks,
Chanel King
___
gmx-users mailing listgmx-users@gromacs.org
http://lists.gromacs.org/mailman/listinfo/gmx-users
Please search the archive at http://www.gromacs.org/search before posting!
Please don't post (un)subscribe requests to the list. Use the
www interface or send it to gmx-users-requ...@gromacs.org.
Can't post? Read http://www.gromacs.org/mailing_lists/users.php




___
gmx-users mailing listgmx-users@gromacs.org
http://lists.gromacs.org/mailman/listinfo/gmx-users
Please search the archive at http://www.gromacs.org/search before posting!
Please don't post (un)subscribe requests to the list. Use the 
www interface or send it to gmx-users-requ...@gromacs.org.

Can't post? Read http://www.gromacs.org/mailing_lists/users.php



--


Justin A. Lemkul
Ph.D. Candidate
ICTAS Doctoral Scholar
Department of Biochemistry
Virginia Tech
Blacksburg, VA
jalemkul[at]vt.edu | (540) 231-9080
http://www.bevanlab.biochem.vt.edu/Pages/Personal/justin


___
gmx-users mailing listgmx-users@gromacs.org
http://lists.gromacs.org/mailman/listinfo/gmx-users
Please search the archive at http://www.gromacs.org/search before posting!
Please don't post (un)subscribe requests to the list. Use the 
www interface or send it to gmx-users-requ...@gromacs.org.

Can't post? Read http://www.gromacs.org/mailing_lists/users.php


Re: [gmx-users] PRODRG

2009-10-07 Thread jorge_quintero
Hello Chanel

Could you send a copy of the PDB file.  I think that the error is related
with label atoms included in each force fiel parameter.

See you.

> Hello,
>  I have recently made a pdb file using the Dundee PRODRG server.
> However, when I try to use this pdb in gromacs, I receive an error message
> that states: "DRG is not in the topology database."  I have tried to use
> the
> available tutorial to solve this issue, but with not much success.  Could
> anyone give me a step by step procedure so that I can use the pdb I have
> made
> using PRODRG?
>
> Thanks,
> Chanel King
> ___
> gmx-users mailing listgmx-users@gromacs.org
> http://lists.gromacs.org/mailman/listinfo/gmx-users
> Please search the archive at http://www.gromacs.org/search before posting!
> Please don't post (un)subscribe requests to the list. Use the
> www interface or send it to gmx-users-requ...@gromacs.org.
> Can't post? Read http://www.gromacs.org/mailing_lists/users.php
>


___
gmx-users mailing listgmx-users@gromacs.org
http://lists.gromacs.org/mailman/listinfo/gmx-users
Please search the archive at http://www.gromacs.org/search before posting!
Please don't post (un)subscribe requests to the list. Use the 
www interface or send it to gmx-users-requ...@gromacs.org.
Can't post? Read http://www.gromacs.org/mailing_lists/users.php


Re: [gmx-users] PRODRG

2009-10-07 Thread Justin A. Lemkul



Smith, Chanel Chonda wrote:

Hello,
 I have recently made a pdb file using the Dundee PRODRG server.
However, when I try to use this pdb in gromacs, I receive an error message
that states: "DRG is not in the topology database."  I have tried to use the
available tutorial to solve this issue, but with not much success.  Could
anyone give me a step by step procedure so that I can use the pdb I have made
using PRODRG?
 


The purpose of PRODRG is to generate the topology, such that you don't have to 
pass it through pdb2gmx.  If the molecule doesn't exist in the force field .rtp 
file, then you will get this error.  John Kerrigan's tutorial explains how to 
use the PRODRG topology within your system .top quite clearly.


Be advised that the charges and charge groups produced by PRODRG are often 
unsatisfactory, requiring manual modification and validation.


-Justin


Thanks,
Chanel King
___
gmx-users mailing listgmx-users@gromacs.org
http://lists.gromacs.org/mailman/listinfo/gmx-users
Please search the archive at http://www.gromacs.org/search before posting!
Please don't post (un)subscribe requests to the list. Use the 
www interface or send it to gmx-users-requ...@gromacs.org.

Can't post? Read http://www.gromacs.org/mailing_lists/users.php



--


Justin A. Lemkul
Ph.D. Candidate
ICTAS Doctoral Scholar
Department of Biochemistry
Virginia Tech
Blacksburg, VA
jalemkul[at]vt.edu | (540) 231-9080
http://www.bevanlab.biochem.vt.edu/Pages/Personal/justin


___
gmx-users mailing listgmx-users@gromacs.org
http://lists.gromacs.org/mailman/listinfo/gmx-users
Please search the archive at http://www.gromacs.org/search before posting!
Please don't post (un)subscribe requests to the list. Use the 
www interface or send it to gmx-users-requ...@gromacs.org.

Can't post? Read http://www.gromacs.org/mailing_lists/users.php


[gmx-users] PRODRG

2009-10-07 Thread Smith, Chanel Chonda
Hello,
 I have recently made a pdb file using the Dundee PRODRG server.
However, when I try to use this pdb in gromacs, I receive an error message
that states: "DRG is not in the topology database."  I have tried to use the
available tutorial to solve this issue, but with not much success.  Could
anyone give me a step by step procedure so that I can use the pdb I have made
using PRODRG?
 
Thanks,
Chanel King
___
gmx-users mailing listgmx-users@gromacs.org
http://lists.gromacs.org/mailman/listinfo/gmx-users
Please search the archive at http://www.gromacs.org/search before posting!
Please don't post (un)subscribe requests to the list. Use the
www interface or send it to gmx-users-requ...@gromacs.org.
Can't post? Read http://www.gromacs.org/mailing_lists/users.php


Re: [gmx-users] Extended structure for Estrogen receptor

2009-10-07 Thread Pradip Biswas
Justin,
I came to this board as a last resource, after failing in my search.

Pradip

On Wed, Oct 7, 2009 at 9:09 AM, Justin A. Lemkul  wrote:

>
>
> Pradip Biswas wrote:
>
>> Dear All,
>>
>> All the estrogen receptor alpha structure that I found on PDB repository,
>> contain mainly the ligand binding domain (residues 301-550). I am looking
>> for a structure that has the hinge region (240-300) as well. Does anyone
>> have any extended structure (even if it is an engineered one) of Estrogen
>> Receptor Alpha which contains the ligand binding domain plus the hinge
>> region residues: 240-550 ??
>>
>
> Search the literature.  This question is unrelated to Gromacs.
>
> -Justin
>
>  thanks in advance,
>> pb.
>>
>>
>> 
>>
>> ___
>> gmx-users mailing listgmx-users@gromacs.org
>> http://lists.gromacs.org/mailman/listinfo/gmx-users
>> Please search the archive at http://www.gromacs.org/search before
>> posting!
>> Please don't post (un)subscribe requests to the list. Use the www
>> interface or send it to gmx-users-requ...@gromacs.org.
>> Can't post? Read http://www.gromacs.org/mailing_lists/users.php
>>
>
> --
> 
>
> Justin A. Lemkul
> Ph.D. Candidate
> ICTAS Doctoral Scholar
> Department of Biochemistry
> Virginia Tech
> Blacksburg, VA
> jalemkul[at]vt.edu | (540) 231-9080
> http://www.bevanlab.biochem.vt.edu/Pages/Personal/justin
>
> 
> ___
> gmx-users mailing listgmx-users@gromacs.org
> http://lists.gromacs.org/mailman/listinfo/gmx-users
> Please search the archive at http://www.gromacs.org/search before posting!
> Please don't post (un)subscribe requests to the list. Use the www interface
> or send it to gmx-users-requ...@gromacs.org.
> Can't post? Read http://www.gromacs.org/mailing_lists/users.php
>



-- 
Pradip K Biswas, PhD
Asst. Professor, Department of Physics,
Computational Bioengineering and Nanoscience Group
Tougaloo College, MS 39174
___
gmx-users mailing listgmx-users@gromacs.org
http://lists.gromacs.org/mailman/listinfo/gmx-users
Please search the archive at http://www.gromacs.org/search before posting!
Please don't post (un)subscribe requests to the list. Use the 
www interface or send it to gmx-users-requ...@gromacs.org.
Can't post? Read http://www.gromacs.org/mailing_lists/users.php

[gmx-users] Distance restraint problem with 4.0.5 version

2009-10-07 Thread Alexandre Suman de Araujo

Hi Gmxers

Some months ago I performed some simulations, with GMX 3.3.3 version, 
where I apply distance restraint between water oxygen atoms and a static 
dummy site located at the center of a sphere to keep waters inside this 
sphere. After performed an upgrade to GMX 4.0.5 version this distance 
restraint scheme stopped to work.
I read distance restraint material in 4.0.5 paper manual and I can't 
find any modification reported there.


Looking at gmxdump output from tpr's generated with grompp from 3.3.3 
and 4.0.5 versions and the same .mdp, .gro and .top files, I had this:


3.3.3 version: grep -i disre gmxdump3.txt |more

  disre= Simple
  disre_weighting  = Conservative
  disre_mixed  = FALSE
  nstdisreout  = 100
functype[468]=DISRES, label=   0, type=1, low= 0.e+00, 
up1= 3.e+00, up2= 9.3962e+00, fac= 1.e+00)
functype[469]=DISRES, label=   1, type=1, low= 0.e+00, 
up1= 3.e+00, up2= 9.3962e+00, fac= 1.e+00)
functype[470]=DISRES, label=   2, type=1, low= 0.e+00, 
up1= 3.e+00, up2= 9.3962e+00, fac= 1.e+00)
functype[471]=DISRES, label=   3, type=1, low= 0.e+00, 
up1= 3.e+00, up2= 9.3962e+00, fac= 1.e+00)
functype[472]=DISRES, label=   4, type=1, low= 0.e+00, 
up1= 3.e+00, up2= 9.3962e+00, fac= 1.e+00)
functype[473]=DISRES, label=   5, type=1, low= 0.e+00, 
up1= 3.e+00, up2= 9.3962e+00, fac= 1.e+00)

.
.
.
functype[724]=DISRES, label= 256, type=1, low= 0.e+00, 
up1= 3.e+00, up2= 9.3962e+00, fac= 1.e+00)
functype[725]=DISRES, label= 257, type=1, low= 0.e+00, 
up1= 3.e+00, up2= 9.3962e+00, fac= 1.e+00)
functype[726]=DISRES, label= 258, type=1, low= 0.e+00, 
up1= 3.e+00, up2= 9.3962e+00, fac= 1.e+00)

   0 type=468 (DISRES) 389 392
   1 type=469 (DISRES) 393 396
   2 type=470 (DISRES) 397 400
   3 type=471 (DISRES) 401 404
   4 type=472 (DISRES) 405 408
   5 type=473 (DISRES) 409 412
   .
   .
   .
   256 type=724 (DISRES) 1413 1416
   257 type=725 (DISRES) 1417 1420
   258 type=726 (DISRES) 1421 1424

4.0.5 version: grep -i disre gmxdump4.txt

  disre= Simple
  disre_weighting  = Conservative
  disre_mixed  = FALSE
  nstdisreout  = 100
functype[470]=DISRES, label=   0, type=1, low= 0.e+00, 
up1= 3.e+00, up2= 9.3962e+00, fac= 1.e+00)

   0 type=470 (DISRES) 0 3


It is clear that 3.3.3 version is applying the constraints to all waters 
(what is correct) and 4.0.5 to just one. However, I can't find a way to 
fix this.


Does anyone could help me with this issue? Are there modifications in 
distance restraint algorithm in 4.0.5 version compared with 3.3.3 
version? Or this can be a bug?


Thanks

--
**
Alexandre Suman de Araujo*
Faculdade de Ciências Farmacêuticas de Ribeirão Preto*
Universidade de São Paulo*
Dep. de Física e Química *
Grupo de Física Biológica * e-mail: asara...@fcfrp.usp.br*
Av. do Café, s/n° * e-mail: ale.su...@gmail.com  *
CEP: 14040-903* Phone: +55 (16) 3602-4172*
Ribeirão Preto, SP, Brasil* Phone: +55 (16) 3602-4222*
** 


___
gmx-users mailing listgmx-users@gromacs.org
http://lists.gromacs.org/mailman/listinfo/gmx-users
Please search the archive at http://www.gromacs.org/search before posting!
Please don't post (un)subscribe requests to the list. Use the 
www interface or send it to gmx-users-requ...@gromacs.org.

Can't post? Read http://www.gromacs.org/mailing_lists/users.php


Re: [gmx-users] Extended structure for Estrogen receptor

2009-10-07 Thread Justin A. Lemkul



Pradip Biswas wrote:

Dear All,

All the estrogen receptor alpha structure that I found on PDB 
repository, contain mainly the ligand binding domain (residues 301-550). 
I am looking for a structure that has the hinge region (240-300) as 
well. Does anyone have any extended structure (even if it is an 
engineered one) of Estrogen Receptor Alpha which contains the ligand 
binding domain plus the hinge region residues: 240-550 ??   



Search the literature.  This question is unrelated to Gromacs.

-Justin


thanks in advance,
pb.




___
gmx-users mailing listgmx-users@gromacs.org
http://lists.gromacs.org/mailman/listinfo/gmx-users
Please search the archive at http://www.gromacs.org/search before posting!
Please don't post (un)subscribe requests to the list. Use the 
www interface or send it to gmx-users-requ...@gromacs.org.

Can't post? Read http://www.gromacs.org/mailing_lists/users.php


--


Justin A. Lemkul
Ph.D. Candidate
ICTAS Doctoral Scholar
Department of Biochemistry
Virginia Tech
Blacksburg, VA
jalemkul[at]vt.edu | (540) 231-9080
http://www.bevanlab.biochem.vt.edu/Pages/Personal/justin


___
gmx-users mailing listgmx-users@gromacs.org
http://lists.gromacs.org/mailman/listinfo/gmx-users
Please search the archive at http://www.gromacs.org/search before posting!
Please don't post (un)subscribe requests to the list. Use the 
www interface or send it to gmx-users-requ...@gromacs.org.

Can't post? Read http://www.gromacs.org/mailing_lists/users.php


[gmx-users] Extended structure for Estrogen receptor

2009-10-07 Thread Pradip Biswas
Dear All,
All the estrogen receptor alpha structure that I found on PDB repository,
contain mainly the ligand binding domain (residues 301-550). I am looking
for a structure that has the hinge region (240-300) as well. Does anyone
have any extended structure (even if it is an engineered one) of Estrogen
Receptor Alpha which contains the ligand binding domain plus the hinge
region residues: 240-550 ??

thanks in advance,
pb.
___
gmx-users mailing listgmx-users@gromacs.org
http://lists.gromacs.org/mailman/listinfo/gmx-users
Please search the archive at http://www.gromacs.org/search before posting!
Please don't post (un)subscribe requests to the list. Use the 
www interface or send it to gmx-users-requ...@gromacs.org.
Can't post? Read http://www.gromacs.org/mailing_lists/users.php

RE: [gmx-users] vsites and lincs-order

2009-10-07 Thread Berk Hess



> Date: Wed, 7 Oct 2009 15:51:00 +0200
> From: joc...@xray.bmc.uu.se
> To: gmx-users@gromacs.org
> Subject: Re: [gmx-users] vsites and lincs-order
> 
> All right, thanks!
> 
> Since you mention energy conservation, maybe it would be worth adding 
> notes or a warnings into pdb2gmx if the time step is large. First, if 
> lincs-order is 4 (instead of 6) and second (more important), if nstlist 
> is not reduced with increasing dt. I strongly feel that most people who 
> use vsites forget to reduce nstlist accordingly.
> 

grompp you mean, I guess?

> I have played a bit with timestep and nstlist and checked the energy 
> drift - apparently neighbor searching should be done at least every 
> 20fs. With less frequent neighbor searching the energy drift increases 
> drastically (with PME and LJ-cutoff, what people typically use). Now 
> imagine a time step of 5fs, with the standard nstlist=10, yields 
> neighbor searching only every 50fs. In my test system that increased the 
> energy drift by a factor of 10-20 compared to the case where neighbor 
> searching was done every 20fs.

We can not do such things.
Gromacs is not a atomistic, protein specific simulations package.
For other systems, masses, etc. such settings could be fine.
Moreover, grompp can not easily check for this specific type of vsite use.
TIP4p, for instance, also contains a vsite.

It has happened that someone put in similar checks in grompp, but this can be
EXTREMELY annoying when you use different systems.

Berk

> 
> Jochen
> 
> 
> Berk Hess wrote:
> > My P-Lincs paper http://dx.doi.org/10.1021/ct700200b shows that
> > with order 6 and a time step of 4 fs you get roughly the same 
> > constraint accuracy
> > and energy conservation as without v-sites and a 2 fs time step.
> > With order 4 and a 4 fs time step the energy drift is 2.2 times higher 
> > than with order 6
> > (not mentioned in the paper). This is not a very large difference.
> >
> > I recall that some time ago I decided to change the default lincs 
> > order to 6
> > (I have not done this yet), but unfortunately now I don't recall what 
> > issue
> > made me decide that the current default accuracy is not high enough.
> >
> > Berk
> >
> > > Date: Wed, 7 Oct 2009 13:16:52 +0200
> > > From: joc...@xray.bmc.uu.se
> > > To: gmx-users@gromacs.org
> > > Subject: [gmx-users] vsites and lincs-order
> > >
> > > Hi,
> > >
> > > the manual suggests lincs-order = 6 when using large time steps (4-5 
> > fs,
> > > with vsites). Has anyone experience how severe that issue is. Has 
> > anyone
> > > observed artefacts with lincs-order=4 and large time steps?
> > >
> > > Thanks a lot,
> > >
> > > Jochen
> > >
> > > --
> > > ---
> > > Dr. Jochen Hub
> > > Molecular Biophysics group
> > > Dept. of Cell & Molecular Biology
> > > Uppsala University. Box 596, 75124 Uppsala, Sweden.
> > > Phone: +46-18-4714451 Fax: +46-18-511755
> > > ---
> > >
> > > ___
> > > gmx-users mailing list gmx-users@gromacs.org
> > > http://lists.gromacs.org/mailman/listinfo/gmx-users
> > > Please search the archive at http://www.gromacs.org/search before 
> > posting!
> > > Please don't post (un)subscribe requests to the list. Use the
> > > www interface or send it to gmx-users-requ...@gromacs.org.
> > > Can't post? Read http://www.gromacs.org/mailing_lists/users.php
> >
> > 
> > What can you do with the new Windows Live? Find out 
> > 
> > 
> >
> > ___
> > gmx-users mailing listgmx-users@gromacs.org
> > http://lists.gromacs.org/mailman/listinfo/gmx-users
> > Please search the archive at http://www.gromacs.org/search before posting!
> > Please don't post (un)subscribe requests to the list. Use the 
> > www interface or send it to gmx-users-requ...@gromacs.org.
> > Can't post? Read http://www.gromacs.org/mailing_lists/users.php
> 
> 
> -- 
> ---
> Dr. Jochen Hub
> Molecular Biophysics group
> Dept. of Cell & Molecular Biology
> Uppsala University. Box 596, 75124 Uppsala, Sweden.
> Phone: +46-18-4714451 Fax: +46-18-511755
> ---
> 
> ___
> gmx-users mailing listgmx-users@gromacs.org
> http://lists.gromacs.org/mailman/listinfo/gmx-users
> Please search the archive at http://www.gromacs.org/search before posting!
> Please don't post (un)subscribe requests to the list. Use the 
> www interface or send it to gmx-users-requ...@gromacs.org.
> Can't post? Read http://www.gromacs.org/mailing_lists/users.php
  
___

Re: [gmx-users] vsites and lincs-order

2009-10-07 Thread Jochen Hub

All right, thanks!

Since you mention energy conservation, maybe it would be worth adding 
notes or a warnings into pdb2gmx if the time step is large. First, if 
lincs-order is 4 (instead of 6) and second (more important), if nstlist 
is not reduced with increasing dt. I strongly feel that most people who 
use vsites forget to reduce nstlist accordingly.


I have played a bit with timestep and nstlist and checked the energy 
drift - apparently neighbor searching should be done at least every 
20fs. With less frequent neighbor searching the energy drift increases 
drastically (with PME and LJ-cutoff, what people typically use). Now 
imagine a time step of 5fs, with the standard nstlist=10, yields 
neighbor searching only every 50fs. In my test system that increased the 
energy drift by a factor of 10-20 compared to the case where neighbor 
searching was done every 20fs.


Jochen


Berk Hess wrote:

My P-Lincs paper http://dx.doi.org/10.1021/ct700200b shows that
with order 6 and a time step of 4 fs you get roughly the same 
constraint accuracy

and energy conservation as without v-sites and a 2 fs time step.
With order 4 and a 4 fs time step the energy drift is 2.2 times higher 
than with order 6

(not mentioned in the paper). This is not a very large difference.

I recall that some time ago I decided to change the default lincs 
order to 6
(I have not done this yet), but unfortunately now I don't recall what 
issue

made me decide that the current default accuracy is not high enough.

Berk

> Date: Wed, 7 Oct 2009 13:16:52 +0200
> From: joc...@xray.bmc.uu.se
> To: gmx-users@gromacs.org
> Subject: [gmx-users] vsites and lincs-order
>
> Hi,
>
> the manual suggests lincs-order = 6 when using large time steps (4-5 
fs,
> with vsites). Has anyone experience how severe that issue is. Has 
anyone

> observed artefacts with lincs-order=4 and large time steps?
>
> Thanks a lot,
>
> Jochen
>
> --
> ---
> Dr. Jochen Hub
> Molecular Biophysics group
> Dept. of Cell & Molecular Biology
> Uppsala University. Box 596, 75124 Uppsala, Sweden.
> Phone: +46-18-4714451 Fax: +46-18-511755
> ---
>
> ___
> gmx-users mailing list gmx-users@gromacs.org
> http://lists.gromacs.org/mailman/listinfo/gmx-users
> Please search the archive at http://www.gromacs.org/search before 
posting!

> Please don't post (un)subscribe requests to the list. Use the
> www interface or send it to gmx-users-requ...@gromacs.org.
> Can't post? Read http://www.gromacs.org/mailing_lists/users.php


What can you do with the new Windows Live? Find out 




___
gmx-users mailing listgmx-users@gromacs.org
http://lists.gromacs.org/mailman/listinfo/gmx-users
Please search the archive at http://www.gromacs.org/search before posting!
Please don't post (un)subscribe requests to the list. Use the 
www interface or send it to gmx-users-requ...@gromacs.org.

Can't post? Read http://www.gromacs.org/mailing_lists/users.php



--
---
Dr. Jochen Hub
Molecular Biophysics group
Dept. of Cell & Molecular Biology
Uppsala University. Box 596, 75124 Uppsala, Sweden.
Phone: +46-18-4714451 Fax: +46-18-511755
---

___
gmx-users mailing listgmx-users@gromacs.org
http://lists.gromacs.org/mailman/listinfo/gmx-users
Please search the archive at http://www.gromacs.org/search before posting!
Please don't post (un)subscribe requests to the list. Use the 
www interface or send it to gmx-users-requ...@gromacs.org.

Can't post? Read http://www.gromacs.org/mailing_lists/users.php


Re: [gmx-users] Add new post-processing features in Gromacs

2009-10-07 Thread Mark Abraham

Tandia, Adama wrote:

Dears,

I would like to know if the details of how to add new post-processing
features in Gromacs had been already discussed or described somewhere.
I have in mind things like angle distribution, structure factor, and
incoherent intermediate scattering function, to name a few, for glassy
materials.


g_angle and g_rdf might already do some of these things.


Does anyone know of a package that could compute those properties from a
Gromacs trajectory files?


No.

Mark
___
gmx-users mailing listgmx-users@gromacs.org
http://lists.gromacs.org/mailman/listinfo/gmx-users
Please search the archive at http://www.gromacs.org/search before posting!
Please don't post (un)subscribe requests to the list. Use the 
www interface or send it to gmx-users-requ...@gromacs.org.

Can't post? Read http://www.gromacs.org/mailing_lists/users.php


RE: [gmx-users] vsites and lincs-order

2009-10-07 Thread Berk Hess

My P-Lincs paper http://dx.doi.org/10.1021/ct700200b shows that
with order 6 and a time step of 4 fs you get roughly the same constraint 
accuracy
and energy conservation as without v-sites and a 2 fs time step.
With order 4 and a 4 fs time step the energy drift is 2.2 times higher than 
with order 6
(not mentioned in the paper). This is not a very large difference.

I recall that some time ago I decided to change the default lincs order to 6
(I have not done this yet), but unfortunately now I don't recall what issue
made me decide that the current default accuracy is not high enough.

Berk

> Date: Wed, 7 Oct 2009 13:16:52 +0200
> From: joc...@xray.bmc.uu.se
> To: gmx-users@gromacs.org
> Subject: [gmx-users] vsites and lincs-order
> 
> Hi,
> 
> the manual suggests lincs-order = 6 when using large time steps (4-5 fs, 
> with vsites). Has anyone experience how severe that issue is. Has anyone 
> observed artefacts with lincs-order=4 and large time steps?
> 
> Thanks a lot,
> 
> Jochen
> 
> -- 
> ---
> Dr. Jochen Hub
> Molecular Biophysics group
> Dept. of Cell & Molecular Biology
> Uppsala University. Box 596, 75124 Uppsala, Sweden.
> Phone: +46-18-4714451 Fax: +46-18-511755
> ---
> 
> ___
> gmx-users mailing listgmx-users@gromacs.org
> http://lists.gromacs.org/mailman/listinfo/gmx-users
> Please search the archive at http://www.gromacs.org/search before posting!
> Please don't post (un)subscribe requests to the list. Use the 
> www interface or send it to gmx-users-requ...@gromacs.org.
> Can't post? Read http://www.gromacs.org/mailing_lists/users.php
  
_
What can you do with the new Windows Live? Find out
http://www.microsoft.com/windows/windowslive/default.aspx___
gmx-users mailing listgmx-users@gromacs.org
http://lists.gromacs.org/mailman/listinfo/gmx-users
Please search the archive at http://www.gromacs.org/search before posting!
Please don't post (un)subscribe requests to the list. Use the 
www interface or send it to gmx-users-requ...@gromacs.org.
Can't post? Read http://www.gromacs.org/mailing_lists/users.php

[gmx-users] vsites and lincs-order

2009-10-07 Thread Jochen Hub

Hi,

the manual suggests lincs-order = 6 when using large time steps (4-5 fs, 
with vsites). Has anyone experience how severe that issue is. Has anyone 
observed artefacts with lincs-order=4 and large time steps?


Thanks a lot,

Jochen

--
---
Dr. Jochen Hub
Molecular Biophysics group
Dept. of Cell & Molecular Biology
Uppsala University. Box 596, 75124 Uppsala, Sweden.
Phone: +46-18-4714451 Fax: +46-18-511755
---

___
gmx-users mailing listgmx-users@gromacs.org
http://lists.gromacs.org/mailman/listinfo/gmx-users
Please search the archive at http://www.gromacs.org/search before posting!
Please don't post (un)subscribe requests to the list. Use the 
www interface or send it to gmx-users-requ...@gromacs.org.

Can't post? Read http://www.gromacs.org/mailing_lists/users.php


Re: [gmx-users] Problem in equilibration for membrane system

2009-10-07 Thread Justin A. Lemkul



Bing Bing wrote:
My protein complex was minimized initially before putting into the 
lipid. POPC is also a preequilibrated structure from Tielemen website. I 
don't quite understand on what you meant by "the protein is not happy 
with the restraint". The system went well through out the nvt, npt with 


I think what Mark was getting as was if you're restraining some unusual or 
inappropriate geometry, you can get nasty results when you turn off restraints. 
 I don't know that this is the case in your situation.


restraint on protein Once it move to md production run ( which is 
without restraint on protein), error occurred (range checking error). 
The reason i'm using the version3.3.2 is because the forcefield i'm 
using at the moment only support version3.3.2.  Is my problem cause by 


I don't understand this statement.  The force field you mentioned is most 
certainly supported in the newest version of Gromacs, in fact my tutorial was 
written for the version 4.0.x series.  Please also see the advanced 
troubleshooting page I provide; you may find some ideas there.


Certainly, watch the trajectory and see where things are going wrong to deduce 
the nature of the issue.


-Justin


bug of the older version? is there any bugfix fot this? please
help.

regards,
Bing


On Wed, Oct 7, 2009 at 4:15 PM, Mark Abraham > wrote:


Bing Bing wrote:

Dear All,
I'm running the protein complex with POPC system. NVT (100ps)
and NPT (200ps) were done with restraint on the protein follow
by 1000 ps of NPT without restraint on protein. The trajectory
was checked in term of pressure, potential, area per lipid
were checked and everything seems ok (potential almost
plateu, area per lipid close to experimental value). I proceeded
with md (similar to Justin's tutorial) for 1 ns. The simulation
stop around 800ps with the range checking error.


OK so look at the collective variables, and/or trajectory to find
evidence of the problem. The most likely explanation is that your
protein was unhappy in the restraints, of course. Perhaps you should
try some vacuum EM before you embed it in the lipid, or some such.


I'm using gromacs 3.3.2, with GROMOS9653a6 forcefield. I tired
to be more gentle while releasing the restraint.Different
stepwise of releasing the restraint during NPT were tried and
followed by production MD, all the simulation stop or i shall
said it crashed around ~800ps with the same error (range
checking error). I'm out of idea what is the problem here.
Please advice.


Range checking is just one of many things that can be the first
thing to break when there's an underlying systemic problem.

Also, unless you need scientific continuity, 3.3.2 is about 4 years
old... 4.0.5 will be much faster!

Mark
___
gmx-users mailing listgmx-users@gromacs.org

http://lists.gromacs.org/mailman/listinfo/gmx-users
Please search the archive at http://www.gromacs.org/search before
posting!
Please don't post (un)subscribe requests to the list. Use the www
interface or send it to gmx-users-requ...@gromacs.org
.
Can't post? Read http://www.gromacs.org/mailing_lists/users.php





___
gmx-users mailing listgmx-users@gromacs.org
http://lists.gromacs.org/mailman/listinfo/gmx-users
Please search the archive at http://www.gromacs.org/search before posting!
Please don't post (un)subscribe requests to the list. Use the 
www interface or send it to gmx-users-requ...@gromacs.org.

Can't post? Read http://www.gromacs.org/mailing_lists/users.php


--


Justin A. Lemkul
Ph.D. Candidate
ICTAS Doctoral Scholar
Department of Biochemistry
Virginia Tech
Blacksburg, VA
jalemkul[at]vt.edu | (540) 231-9080
http://www.bevanlab.biochem.vt.edu/Pages/Personal/justin


___
gmx-users mailing listgmx-users@gromacs.org
http://lists.gromacs.org/mailman/listinfo/gmx-users
Please search the archive at http://www.gromacs.org/search before posting!
Please don't post (un)subscribe requests to the list. Use the 
www interface or send it to gmx-users-requ...@gromacs.org.

Can't post? Read http://www.gromacs.org/mailing_lists/users.php


Re: [gmx-users] step 0Segmentation fault

2009-10-07 Thread Justin A. Lemkul



ram bio wrote:

Dear Justin,

Thanks for the options suggested.

I have used -princ and rotate 0 0 90 options with editconf and was
able to place the protein vertically and at the centre of the DPPC
bilayer (128 + 3655) from the site provided in the tutorial, but still
the part of protein is outside the DPPC layer (examined system.gro
(protein newbox +dppc128.gr0) in VMD). The commands used were



This isn't necessarily a problem.  So your protein sticks out - that will 
probably be the case in most membrane protein systems.



editconf -f protein_processed.gro -o protein_newbox.gro  -c -box
6.41840 6.44350 6.59650 -rotate 0 0 90



Depending on the degree to which the protein is protruding out of the bilayer, 
you may have to adjust the z dimension of the box, as I've suggested before.



cat protein_newbox.gro dppc128.gro > system.gro


so now, i tried to increase the size of the box by redoing it and
using the command

editconf -f protein_processed.gro -o protein_newbox.gro -c -box
6.41840 6.44350 6.59650 -d 1.0 -rotate 0 0 90 -bt cubic



I don't know what effect this will have.  It is contradictory to specify a 
non-cubic box by assigning the vectors explicitly, then using -bt cubic.  I 
doubt you're getting what you think you are, but I don't know which option has 
precendence, -box or -bt.  Either way, you're not doing anything to increase the 
box size.  Also note that the box size in dppc128.gro would have to be similarly 
manipulated to actually get a larger box.



cat protein_newbox.gro dppc128.gro > system1.gro

I also used various box types like dodecahedron , octahedron but was


These boxes make no sense for a square cross-section of a membrane.


unable increase the size of the box and fix the protein in the centre
of the dppcbilayer (128 + 3655 H2O). Please suggest me how to correct
my command so that all the protein lies in the dppcbilayer and is in
the  centre and can proceed to inflategro steps, also do i need to use
a dppclayer with more lipids like 256 lipid bilayer.


Well, using editconf properly will help.  Define a proper box with -box, don't 
just keep copying the one I use in the tutorial.  The x and y components may 
stay the same, but you may have to increase the z vector.


The same process applies with any bilayer, even if it has 256 lipids.

-Justin



Thanks,

Ram

On Tue, Oct 6, 2009 at 5:39 PM, Justin A. Lemkul  wrote:


ram bio wrote:

Dear Justin,

As suggested, when i reexamined the system.gro (protein_newbox +
dppc128) one of the ends of the problem (as i think) i.e. few
aminoacids of protein were beyond the water surface of the bilayer,
probably this may be the reason for the presence of water molecules to
the side of the bilayer when solvated and also lack of minimization
during inflategro step. One more thing, I am inserting the protein in

Indeed, if your system doesn't actually fit within the unit cell you've
defined, you're in for trouble.  Always look at your output!


lipid bilayer by orienting it using your KALP peptide (.pdb) as
reference in VMD and later using editconf to centre the protein using
option i.e. -c -box 6.41840 6.44350 6.59650, otherwise if i just use
editconf command, it is orienting horizontally instead of vertically,

I don't fully understand what you're doing.  You also may not be able to use
the exact same box that I defined in the tutorial (the original DPPC box).
 If you've got pieces of your protein protruding out of the unit cell, then
you'll need to define a suitably-sized box.  Horizontal vs. vertical issues
can be solved by editconf -rotate.

-Justin


i can do the orientation manually using vmd but it is difficult and
iam unable to orient it exactly in the centre and vertical in the dppc
bilayer.

Please suggest some corrections as I am going to reorient and position
it in the bilayer and redo  the inflategro procedure.

Thanks

Ram

On Tue, Oct 6, 2009 at 4:42 PM, Justin A. Lemkul  wrote:

ram bio wrote:

Dear Justin,

Thanks for the advice. I am using the DPPC 128 lipid bilayer from D.
Peter Tieleman website, and the nvt.mdp file and the nvt.log files are
as follows as in your tutorial:




Constraining the starting coordinates (step 0)

Constraining the coordinates at t0-dt (step 0)
RMS relative constraint deviation after constraining: 9.42e-04
Initial temperature: 503.557 K

Started mdrun on node 0 Thu Jun 25 11:30:30 2009

 Step   Time Lambda
00.00.0

Grid: 9 x 9 x 9 cells
Large VCM(group Protein_DPPC):-50.08205, 97.99061,
16.32530, Temp-cm:  2.50669e+05
Long Range LJ corr.:  2.0307e-03
Long Range LJ corr.: Epot   -1862.02, Pres:-184.12, Vir:1862.02
 Energies (kJ/mol)
Angle   G96AngleProper Dih. Ryckaert-Bell.  Improper
Dih.
  1.48814e+048.19090e+038.43857e+036.38969e+03
 2.93352e+03
LJ-14 Coulomb-14LJ (SR)  Disper. corr.   Coulomb
(SR)
  4.09831e+035.49186e+047.87055e+09   -1.86202e+03
-1.4

Re: [gmx-users] S-type Hydrogen bond correlation function

2009-10-07 Thread David van der Spoel

Ramachandran G wrote:

I have used the option as follows:
   g_hbond -f  file.trr -s file.tpr -n file.ndx -ac output.xvg

-life koko.xvg



To find the continious HB-correlation function, what option should i
need to use.
Thank you for your help.
Rama

On Tue, Oct 6, 2009 at 11:40 PM, David van der Spoel
 wrote:

Ramachandran G wrote:

Thank you for the reference. But still i like check it out for my
system.  But still i don't know how to get 'S' type hydrogen bonding.
I am pasting my screen output below:

You need to pass the -life option.



-
Checking for overlap in atoms between plane-B and SOL
Calculating hydrogen bonds between plane-B (434 atoms) and SOL (33036
atoms)
Found 6650 donors and 19904 acceptors
trn version: GMX_trn_file (single precision)
Reading frame   0 time0.000
Will do grid-seach on 16x16x16 grid, rcut=0.35
Last frame  3 time  300.000
Found 3304 different hydrogen bonds in trajectory
Found 7135 different atom-pairs within hydrogen bonding distance
Merging hbonds with Acceptor and Donor swapped
6650/6650
- Reduced number of hbonds from 3304 to 3151
- Reduced number of distances from 7135 to 7135

Back Off! I just backed up hbnum.xvg to ./#hbnum.xvg.2#
Average number of hbonds per timeframe 98.421 out of 6.61808e+07 possible
ACF 3151/3151
Normalization for c(t) = 0.0101617 for gh(t) = 3.38722e-07

WARNING: Correlation function is probably not long enough
because the standard deviation in the tail of C(t) > 0.001
Tail value (average C(t) over second half of acf): 0.124513 +/- 0.0254161
  Hydrogen bond thermodynamics at T = 298.15 K
--
Type  Rate (1/ps)  Time (ps)  DG (kJ/mol)
Forward 0.035 28.458  12.829
Backward   -0.009   -111.540-666.000
One-way 0.074 13.435  10.968
Integral0.047 21.471  12.130
Relaxation  0.047 21.358  12.117

gcq#129: "She's a Good Sheila Bruce" (Monty Python)

---

The hydrogen bond Autocorrelation function output has five columns.
The first columne is the time axis,
i think the second column gives the C(t) other 3 columns i don't
understand. Will you please help me to understand thank you.
Rama

On Tue, Oct 6, 2009 at 3:06 AM, David van der Spoel
 wrote:

Ramachandran G wrote:

C_HB(T)  =  / 

S_HB(T)  =  /


 h(T)  =  1, if a pair of atoms bonded at time T,
=  0, otherwise

 H(T) = 1, if a pair of atoms continously bonded
between time 0 to time T,
= 0, otherwise

S_HB(t) probes the continuous existence of a hydrogen bond, while
C_HB(t) allows for
the reformation of a bond that is broken at some intermediate time. The
former
is thus a strict definition of the hydrogen bond lifetime, while the
latter is more
permissive.

I feel in gromacs, g_hbond does the C_HB(t). But i am not very sure
about that. Anybody knows
please help me. Thank you.

This is correct. However, the S_HB definition is not very useful, see my
paper J. Phys. Chem. B. 110 pp. 4393-4398 (2006) for an explanation. The
uninterrupted life time is computed by g_hbond however. Check the screen
output.


Rama

On Tue, Oct 6, 2009 at 2:37 AM, David van der Spoel
 wrote:

Ramachandran G wrote:

Dear gromacs users:
I like to know whether gromacs will calculate S-type hydrogen
bond correlation function?
If it so how it can be done? Thank you.

What does that mean?



Rama
___
gmx-users mailing listgmx-users@gromacs.org
http://lists.gromacs.org/mailman/listinfo/gmx-users
Please search the archive at http://www.gromacs.org/search before
posting!
Please don't post (un)subscribe requests to the list. Use the www
interface or send it to gmx-users-requ...@gromacs.org.
Can't post? Read http://www.gromacs.org/mailing_lists/users.php

--
David van der Spoel, Ph.D., Professor of Biology
Molec. Biophys. group, Dept. of Cell & Molec. Biol., Uppsala
University.
Box 596, 75124 Uppsala, Sweden. Phone:  +46184714205. Fax: +4618511755.
sp...@xray.bmc.uu.sesp...@gromacs.org   http://folding.bmc.uu.se
___
gmx-users mailing listgmx-users@gromacs.org
http://lists.gromacs.org/mailman/listinfo/gmx-users
Please search the archive at http://www.gromacs.org/search before
posting!
Please don't post (un)subscribe requests to the list. Use the www
interface
or send it to gmx-users-requ...@gromacs.org.
Can't post? Read http://www.gromacs.org/mailing_lists/users.php




--
David van der Spoel, Ph.D., Professor of Biology
Molec. Biophys. group, Dept. of Cell & Molec. Biol., Uppsala University.
Box 596, 75124 Uppsala, Sweden. Phone

Re: [gmx-users] step 0Segmentation fault

2009-10-07 Thread ram bio
Dear Justin,

Thanks for the options suggested.

I have used -princ and rotate 0 0 90 options with editconf and was
able to place the protein vertically and at the centre of the DPPC
bilayer (128 + 3655) from the site provided in the tutorial, but still
the part of protein is outside the DPPC layer (examined system.gro
(protein newbox +dppc128.gr0) in VMD). The commands used were

editconf -f protein_processed.gro -o protein_newbox.gro  -c -box
6.41840 6.44350 6.59650 -rotate 0 0 90

cat protein_newbox.gro dppc128.gro > system.gro


so now, i tried to increase the size of the box by redoing it and
using the command

editconf -f protein_processed.gro -o protein_newbox.gro -c -box
6.41840 6.44350 6.59650 -d 1.0 -rotate 0 0 90 -bt cubic

cat protein_newbox.gro dppc128.gro > system1.gro

I also used various box types like dodecahedron , octahedron but was
unable increase the size of the box and fix the protein in the centre
of the dppcbilayer (128 + 3655 H2O). Please suggest me how to correct
my command so that all the protein lies in the dppcbilayer and is in
the  centre and can proceed to inflategro steps, also do i need to use
a dppclayer with more lipids like 256 lipid bilayer.

Thanks,

Ram

On Tue, Oct 6, 2009 at 5:39 PM, Justin A. Lemkul  wrote:
>
>
> ram bio wrote:
>>
>> Dear Justin,
>>
>> As suggested, when i reexamined the system.gro (protein_newbox +
>> dppc128) one of the ends of the problem (as i think) i.e. few
>> aminoacids of protein were beyond the water surface of the bilayer,
>> probably this may be the reason for the presence of water molecules to
>> the side of the bilayer when solvated and also lack of minimization
>> during inflategro step. One more thing, I am inserting the protein in
>
> Indeed, if your system doesn't actually fit within the unit cell you've
> defined, you're in for trouble.  Always look at your output!
>
>> lipid bilayer by orienting it using your KALP peptide (.pdb) as
>> reference in VMD and later using editconf to centre the protein using
>> option i.e. -c -box 6.41840 6.44350 6.59650, otherwise if i just use
>> editconf command, it is orienting horizontally instead of vertically,
>
> I don't fully understand what you're doing.  You also may not be able to use
> the exact same box that I defined in the tutorial (the original DPPC box).
>  If you've got pieces of your protein protruding out of the unit cell, then
> you'll need to define a suitably-sized box.  Horizontal vs. vertical issues
> can be solved by editconf -rotate.
>
> -Justin
>
>> i can do the orientation manually using vmd but it is difficult and
>> iam unable to orient it exactly in the centre and vertical in the dppc
>> bilayer.
>>
>> Please suggest some corrections as I am going to reorient and position
>> it in the bilayer and redo  the inflategro procedure.
>>
>> Thanks
>>
>> Ram
>>
>> On Tue, Oct 6, 2009 at 4:42 PM, Justin A. Lemkul  wrote:
>>>
>>> ram bio wrote:

 Dear Justin,

 Thanks for the advice. I am using the DPPC 128 lipid bilayer from D.
 Peter Tieleman website, and the nvt.mdp file and the nvt.log files are
 as follows as in your tutorial:
>>>
>>> 
>>>
 Constraining the starting coordinates (step 0)

 Constraining the coordinates at t0-dt (step 0)
 RMS relative constraint deviation after constraining: 9.42e-04
 Initial temperature: 503.557 K

 Started mdrun on node 0 Thu Jun 25 11:30:30 2009

  Step   Time Lambda
 00.00.0

 Grid: 9 x 9 x 9 cells
 Large VCM(group Protein_DPPC):-50.08205, 97.99061,
 16.32530, Temp-cm:  2.50669e+05
 Long Range LJ corr.:  2.0307e-03
 Long Range LJ corr.: Epot   -1862.02, Pres:-184.12, Vir:1862.02
  Energies (kJ/mol)
 Angle   G96AngleProper Dih. Ryckaert-Bell.  Improper
 Dih.
   1.48814e+048.19090e+038.43857e+036.38969e+03
  2.93352e+03
 LJ-14 Coulomb-14LJ (SR)  Disper. corr.   Coulomb
 (SR)
   4.09831e+035.49186e+047.87055e+09   -1.86202e+03
 -1.48414e+05
  Coul. recip. Position Rest.  PotentialKinetic En.   Total
 Energy
  -1.53985e+059.50084e+007.87034e+093.08099e+17
  3.08099e+17
  Conserved En.Temperature Pressure (bar)  Cons. rmsd ()
   3.08099e+172.31982e+151.01551e+167.25066e+04

>>> The information shown here indicates very strongly that you have severe
>>> atomic overlap in your starting structure.  This is probably from the
>>> InflateGRO minimization that did not converge appropriately.  Your
>>> potential
>>> energy is astronomically high, as well as factors like temperature, and
>>> thus
>>> kinetic energy.  The latter are related to trying to constrain an
>>> inappropriate starting structure.
>>>
>>> I would suggest going back to the initial construction stage, figuring
>>> out
>>> why that minimization didn't converge, and start

Re: [gmx-users] Problem in equilibration for membrane system

2009-10-07 Thread Bing Bing
My protein complex was minimized initially before putting into the lipid.
POPC is also a preequilibrated structure from Tielemen website. I don't
quite understand on what you meant by "the protein is not happy with the
restraint". The system went well through out the nvt, npt with restraint on
protein Once it move to md production run ( which is without restraint
on protein), error occurred (range checking error).
The reason i'm using the version3.3.2 is because the forcefield i'm using at
the moment only support version3.3.2.  Is my problem cause by bug of the
older version? is there any bugfix fot this? please
help.

regards,
Bing


On Wed, Oct 7, 2009 at 4:15 PM, Mark Abraham wrote:

> Bing Bing wrote:
>
>> Dear All,
>> I'm running the protein complex with POPC system. NVT (100ps) and NPT
>> (200ps) were done with restraint on the protein follow by 1000 ps of NPT
>> without restraint on protein. The trajectory was checked in term of
>> pressure, potential, area per lipid were checked and everything seems ok
>> (potential almost plateu, area per lipid close to experimental value). I
>> proceeded with md (similar to Justin's tutorial) for 1 ns. The simulation
>> stop around 800ps with the range checking error.
>>
>
> OK so look at the collective variables, and/or trajectory to find evidence
> of the problem. The most likely explanation is that your protein was unhappy
> in the restraints, of course. Perhaps you should try some vacuum EM before
> you embed it in the lipid, or some such.
>
>  I'm using gromacs 3.3.2, with GROMOS9653a6 forcefield. I tired to be more
>> gentle while releasing the restraint.Different stepwise of releasing the
>> restraint during NPT were tried and followed by production MD, all the
>> simulation stop or i shall said it crashed around ~800ps with the same error
>> (range checking error). I'm out of idea what is the problem here. Please
>> advice.
>>
>
> Range checking is just one of many things that can be the first thing to
> break when there's an underlying systemic problem.
>
> Also, unless you need scientific continuity, 3.3.2 is about 4 years old...
> 4.0.5 will be much faster!
>
> Mark
> ___
> gmx-users mailing listgmx-users@gromacs.org
> http://lists.gromacs.org/mailman/listinfo/gmx-users
> Please search the archive at http://www.gromacs.org/search before posting!
> Please don't post (un)subscribe requests to the list. Use the www interface
> or send it to gmx-users-requ...@gromacs.org.
> Can't post? Read http://www.gromacs.org/mailing_lists/users.php
>
___
gmx-users mailing listgmx-users@gromacs.org
http://lists.gromacs.org/mailman/listinfo/gmx-users
Please search the archive at http://www.gromacs.org/search before posting!
Please don't post (un)subscribe requests to the list. Use the 
www interface or send it to gmx-users-requ...@gromacs.org.
Can't post? Read http://www.gromacs.org/mailing_lists/users.php

Re: [gmx-users] Problem in equilibration for membrane system

2009-10-07 Thread Mark Abraham

Bing Bing wrote:

Dear All,
I'm running the protein complex with POPC system. NVT (100ps) and NPT 
(200ps) were done with restraint on the protein follow by 1000 ps of NPT 
without restraint on protein. The trajectory was checked in term of 
pressure, potential, area per lipid were checked and everything 
seems ok (potential almost plateu, area per lipid close to experimental 
value). I proceeded with md (similar to Justin's tutorial) for 1 ns. The 
simulation stop around 800ps with the range checking error.


OK so look at the collective variables, and/or trajectory to find 
evidence of the problem. The most likely explanation is that your 
protein was unhappy in the restraints, of course. Perhaps you should try 
some vacuum EM before you embed it in the lipid, or some such.


I'm using 
gromacs 3.3.2, with GROMOS9653a6 forcefield. I tired to be more gentle 
while releasing the restraint.Different stepwise of releasing the 
restraint during NPT were tried and followed by production MD, all the 
simulation stop or i shall said it crashed around ~800ps with the same 
error (range checking error). I'm out of idea what is the problem here. 
Please advice.


Range checking is just one of many things that can be the first thing to 
break when there's an underlying systemic problem.


Also, unless you need scientific continuity, 3.3.2 is about 4 years 
old... 4.0.5 will be much faster!


Mark
___
gmx-users mailing listgmx-users@gromacs.org
http://lists.gromacs.org/mailman/listinfo/gmx-users
Please search the archive at http://www.gromacs.org/search before posting!
Please don't post (un)subscribe requests to the list. Use the 
www interface or send it to gmx-users-requ...@gromacs.org.

Can't post? Read http://www.gromacs.org/mailing_lists/users.php


Re: [gmx-users] S-type Hydrogen bond correlation function

2009-10-07 Thread Ramachandran G
I have used the option as follows:
   g_hbond -f  file.trr -s file.tpr -n file.ndx -ac output.xvg

To find the continious HB-correlation function, what option should i
need to use.
Thank you for your help.
Rama

On Tue, Oct 6, 2009 at 11:40 PM, David van der Spoel
 wrote:
> Ramachandran G wrote:
>>
>> Thank you for the reference. But still i like check it out for my
>> system.  But still i don't know how to get 'S' type hydrogen bonding.
>> I am pasting my screen output below:
>
> You need to pass the -life option.
>
>>
>>
>> -
>> Checking for overlap in atoms between plane-B and SOL
>> Calculating hydrogen bonds between plane-B (434 atoms) and SOL (33036
>> atoms)
>> Found 6650 donors and 19904 acceptors
>> trn version: GMX_trn_file (single precision)
>> Reading frame       0 time    0.000
>> Will do grid-seach on 16x16x16 grid, rcut=0.35
>> Last frame      3 time  300.000
>> Found 3304 different hydrogen bonds in trajectory
>> Found 7135 different atom-pairs within hydrogen bonding distance
>> Merging hbonds with Acceptor and Donor swapped
>> 6650/6650
>> - Reduced number of hbonds from 3304 to 3151
>> - Reduced number of distances from 7135 to 7135
>>
>> Back Off! I just backed up hbnum.xvg to ./#hbnum.xvg.2#
>> Average number of hbonds per timeframe 98.421 out of 6.61808e+07 possible
>> ACF 3151/3151
>> Normalization for c(t) = 0.0101617 for gh(t) = 3.38722e-07
>>
>> WARNING: Correlation function is probably not long enough
>> because the standard deviation in the tail of C(t) > 0.001
>> Tail value (average C(t) over second half of acf): 0.124513 +/- 0.0254161
>>   Hydrogen bond thermodynamics at T = 298.15 K
>> --
>> Type      Rate (1/ps)  Time (ps)  DG (kJ/mol)
>> Forward         0.035     28.458      12.829
>> Backward       -0.009   -111.540    -666.000
>> One-way         0.074     13.435      10.968
>> Integral        0.047     21.471      12.130
>> Relaxation      0.047     21.358      12.117
>>
>> gcq#129: "She's a Good Sheila Bruce" (Monty Python)
>>
>> ---
>>
>> The hydrogen bond Autocorrelation function output has five columns.
>> The first columne is the time axis,
>> i think the second column gives the C(t) other 3 columns i don't
>> understand. Will you please help me to understand thank you.
>> Rama
>>
>> On Tue, Oct 6, 2009 at 3:06 AM, David van der Spoel
>>  wrote:
>>>
>>> Ramachandran G wrote:

             C_HB(T)  =  / 

             S_HB(T)  =  /


                  h(T)  =  1, if a pair of atoms bonded at time T,
                         =  0, otherwise

                  H(T) = 1, if a pair of atoms continously bonded
 between time 0 to time T,
                         = 0, otherwise

 S_HB(t) probes the continuous existence of a hydrogen bond, while
 C_HB(t) allows for
 the reformation of a bond that is broken at some intermediate time. The
 former
 is thus a strict definition of the hydrogen bond lifetime, while the
 latter is more
 permissive.

 I feel in gromacs, g_hbond does the C_HB(t). But i am not very sure
 about that. Anybody knows
 please help me. Thank you.
>>>
>>> This is correct. However, the S_HB definition is not very useful, see my
>>> paper J. Phys. Chem. B. 110 pp. 4393-4398 (2006) for an explanation. The
>>> uninterrupted life time is computed by g_hbond however. Check the screen
>>> output.
>>>
 Rama

 On Tue, Oct 6, 2009 at 2:37 AM, David van der Spoel
  wrote:
>
> Ramachandran G wrote:
>>
>> Dear gromacs users:
>>     I like to know whether gromacs will calculate S-type hydrogen
>> bond correlation function?
>> If it so how it can be done? Thank you.
>
> What does that mean?
>
>
>> Rama
>> ___
>> gmx-users mailing list    gmx-us...@gromacs.org
>> http://lists.gromacs.org/mailman/listinfo/gmx-users
>> Please search the archive at http://www.gromacs.org/search before
>> posting!
>> Please don't post (un)subscribe requests to the list. Use the www
>> interface or send it to gmx-users-requ...@gromacs.org.
>> Can't post? Read http://www.gromacs.org/mailing_lists/users.php
>
> --
> David van der Spoel, Ph.D., Professor of Biology
> Molec. Biophys. group, Dept. of Cell & Molec. Biol., Uppsala
> University.
> Box 596, 75124 Uppsala, Sweden. Phone:  +46184714205. Fax: +4618511755.
> sp...@xray.bmc.uu.se    sp...@gromacs.org   http://folding.bmc.uu.se
> ___
> gmx-users mailing list    gmx-us...@gromacs.org
> http://lists.gromacs.org

[gmx-users] Problem in equilibration for membrane system

2009-10-07 Thread Bing Bing
Dear All,
I'm running the protein complex with POPC system. NVT (100ps) and NPT
(200ps) were done with restraint on the protein follow by 1000 ps of NPT
without restraint on protein. The trajectory was checked in term of
pressure, potential, area per lipid were checked and everything seems ok
(potential almost plateu, area per lipid close to experimental value). I
proceeded with md (similar to Justin's tutorial) for 1 ns. The simulation
stop around 800ps with the range checking error. I'm using gromacs 3.3.2,
with GROMOS9653a6 forcefield. I tired to be more gentle while releasing the
restraint.Different stepwise of releasing the restraint during NPT were
tried and followed by production MD, all the simulation stop or i shall said
it crashed around ~800ps with the same error (range checking error). I'm out
of idea what is the problem here. Please advice.

thanks in advance.

regards,
Bing
___
gmx-users mailing listgmx-users@gromacs.org
http://lists.gromacs.org/mailman/listinfo/gmx-users
Please search the archive at http://www.gromacs.org/search before posting!
Please don't post (un)subscribe requests to the list. Use the 
www interface or send it to gmx-users-requ...@gromacs.org.
Can't post? Read http://www.gromacs.org/mailing_lists/users.php